You are on page 1of 185

Lecture Notes in Mathematics

Arkansas Tech University


Department of Mathematics

Introductory Notes in Ordinary Differential


Equations for Physical Sciences and
Engineering
Solution Guide
Marcel B. Finan
c
All
Rights Reserved

Contents
1 Basic Terminology

2 Existence and Uniqueness of Solutions to First Order Linear


IVP
15
3 Analytical Solution: The Method of Integrating Factor

21

4 Existence and Uniqueness of Solutions to First Order Nonlinear IVP


33
5 Separable Differential Equations

37

6 Exact Differential Equations

47

7 Substitution Techniques: Bernoulli and Riccati Equations

59

8 Graphical Solution: Direction Field of y 0 = f (t, y)

67

9 Numerical Solutions to ODEs: Eulers Method and its Variants


77
10 Second Order Linear Differential Equations: Existence and
Uniqueness Results
83
11 The General Solution of 2nd Order Linear Homogeneous Equations
87
12 Second Order Linear Homogeneous Equations with Constant
Coefficients: Distinct Characterisitc Roots
95
3

CONTENTS

13 Characteristic Equations with Repeated Roots

103

14 Characteristic Equations with Complex Roots

107

15 Series Solutions of Differential Equations

117

16 The Structure of the General Solution of Linear Nonhomogeneous Equations


129
17 The Method of Variation of Parameters

137

18 The Laplace Transform: Basic Definitions and Results

145

19 Further Studies of Laplace Transform

153

20 The Laplace Transform and the Method of Partial Fractions161


21 Laplace Transforms of Periodic Functions

171

22 Convolution Integrals

181

1 Basic Terminology
Problem 1.1
A car starts from rest and accelerates in a straight line at 1.6 m/sec2 for 10
seconds.
(a) What is its final speed?
(b) How far has it travelled in this time?
Solution.
(a) The velocity is related to acceleration according to the equation
v(t) = v0 + at
where a stands for acceleration. Since v0 = 0 (car starting from rest) and
a = 1.6 m/sec2 we can write
v(t) = 1.6t.
The final speed of the car is |v(10)| = 16 m/sec.
(b) The displacement of the car is given by x(t) = 21 at2 + v0 t + x0 = 0.8t2 .
Thus, the car traveled a distance of x(10) = 80 m
Problem 1.2
An object is thrown upward at time t = 0. After t seconds, its height is
y(t) = 4.9t2 + 7t + 1.5
meters above the ground.
(a) From what height was the object thrown?
(b) What is the initial velocity of the object?
(c) What is the acceleration due to gravity?
5

1 BASIC TERMINOLOGY

Solution.
(a) The initial height is y(0) = 1.5 meters.
(b) The velocity of the object at time t is given by v(t) = y 0 (t) = 9.8t + 7.
The initial velocity of the object is v(0) = 7 m/sec.
(c) The acceleration due to gravity is y 00 (t) = 9.8 m/sec2
Problem 1.3
An object thrown in the air on a planet in a distant galaxy is at height
y(t) = 25t2 + 72t + 40 feet at time t seconds after it is thrown. What is
the acceleration due to gravity on that planet? With what velocity was the
object thrown? From what height?
Solution.
The acceleration due to gravity is y 00 (t) = 50 ft/sec2 . The initial velocity is
v(0) = y 0 (0) = 72 ft/sec. The initial height is y(0) = 40 ft
Problem 1.4
An object is dropped from a 400-foot tower. When does it hit the ground
and how fast is it going at the time of the impact?
Solution.
The height of the object at any time t is given by y(t) = 16t2 + 400. The
object hits the ground when y(t) = 0. Solving the equation 16t2 + 400 = 0
we find t = 5 ft. The velocity of the object at the time of impact is v(5) =
y 0 (5) = 32(5) = 160 ft/sec
Problem 1.5
A ball that is dropped from a window hits the ground in five seconds. How
high is the window? (Give your answer in feet.)
Solution.
The height of the object at any time t is given by y(t) = 16t2 + y0 . We are
told that y(5) = 0. Thus, y0 = 16(5)2 = 400 ft
Problem 1.6
A ball is thrown straight up from ground level and reaches its greatest height
after 5 seconds. Find the initial velocity of the ball and the value of its
maximum height above ground level.

7
Solution.
Let y(t) be the height of the ball above ground level at time t seconds after
it was thrown. We are given that y(0) = 0. We are also told that the ball
reaches its maximum height after 5 seconds at which point the velocity is
zero, i.e., v(5) = 0.
The bodys position is governed by the differential equation y 00 (t) = 32 ft/sec2 .
So y 0 (t) = v(t) = 32t + C1 for some constant C1 . Since v(5) = 0, solving
the equation 32(5) + C1 = 0 for C1 we find C1 = 160. Hence,
y 0 (t) = v(t) = 32t + 160.
Using this equation we have now that the initial velocity of the ball was
v(0) = 160 ft/sec. We still need to find the position of the ball at time
5 seconds (when the ball was at its greatest height). By integrating the
previous equation we find
y(t) = 16t2 + 160t + C2 .
Since the ball was thrown from ground level, we have that y(0) = 0, so C2 = 0
and
y(t) = 16t2 + 160t.
We were told that the maximum height was reached after five seconds, so
the maximum heights value is given by
y(5) = 16(5)2 + 160(5) = 400 ft
Problem 1.7
At time t = 0 an object having mass m is released from rest at a height y0
above the ground. Let g represent the constant gravitational acceleration.
Derive an expression for the impact time (the time at which the object strikes
the ground). What is the velocity with which the object strikes the ground?
Solution.
The motion satisfies the differential equation y 00 = g. Integrating twice and
using the facts that v(0) = 0 and y(0) = y0 we find
1
y(t) = gt2 + y0 .
2
The object strikes the ground when y(t) = 0. Thus, 12 gt2 + y0 = 0. Solving
q
for t we find t = 2yg0 . The velocity with which the object strikes the ground
q
q

2y0
is v( g ) = g( 2yg0 ) = 2gy0

1 BASIC TERMINOLOGY

Problem 1.8
At time t = 0, an object of mass m is released from rest at a height of
252 ft above the floor of an experimental chamber in which gravitational
acceleration has been slightly modified. Assume (instead of the usual value
ft/sec2 , where 
of 32 ft/sec2 ), that the acceleration has the form 32 sin t
4
is a positive constant. In addition, assume that the object strikes the ground
exactly 4 sec after release. Can this information be used to determine the
constant ? If so, determine .
Solution.

). The
The motion of the object satisfies the equationy 00 = (32  sin t
4
t
4
velocity is given by v(t) = 32t ( ) cos
 44 + C. But v(0) = 0 so that
t
4
.
Thus,
v(t)
=
32t

(
)
cos
C = 4
+ .

4
The displacement function is given by
 
 2
t
4
4
2
 sin
y(t) = 16t
+ t + 252.

The object strikes the ground at t = 4 sec. In this case y(4) = 0. Solving for
 we find  = 4
Problem 1.9
Find the order of the following differential equations.
(a) ty 00 + y = t3 .
(b) y 0 + y 2 = 2.
(c) sin y 000 + 3t2 y = 6t.
Solution.
(a) Since the highest derivative appearing in the equation is 2, the order of
the equation is 2.
(b) Order is 1
(c) Order is 3
Problem 1.10
What is the order of the differential equation?
(a) y 0 (t) 1 = 0.
(b) y 00 (t) 1 = 0.
(c) y 00 (t) 2ty(t) = 0.
1
t
(d) y 00 (t)(y 0 (t)) 2 y(t)
= 0.

9
Solution.
(a) First order
(b) , (c), and (d) second order
Problem 1.11
In the equation
u u

= x 2y
x y
identify the independent variable(s) and the dependent variable.
Solution.
u is the depedent variable whereas x and y are the independent variables
Problem 1.12
Classify the following equations as either ordinary or partial.
(a) (y 000 )4 +
(b)

u
x

t2
(y 0 )2 +4

+ y u
=
y

= 0.

yx
.
y+x

(c) y 00 4y = 0.
Solution.
(a) ODE.
(b) PDE.
(c) ODE
Problem 1.13
Solve the equation y 000 (t) 2 = 0 by computing successive antiderivatives.
Solution.
Integrating for the first time we find y 00 (t) = 2t + C1 . Integrating the last
equation we find y 0 (t) = t2 + C1 t + C2 . Integrating for a third and final time
3
2
we get y(t) = t3 + C1 t2 + C2 t + C3
Problem 1.14
Solve the initial-value problem
dy
= 3y(t), y(0) = 50.
dt
What is the domain of the solution?

10

1 BASIC TERMINOLOGY

Solution.
The general solution is of the form y(t) = Ce3t . Since y(0) = 50, we find
50 = Ce30 , and so C = 50. The solution is y(t) = 50e3t . The domain is the
set of all real numbers
Problem 1.15
For what real value(s) of is y = cos t a solution of the equation y 00 +9y = 0?
Solution.
Finding the first and second derivatives, we find that y 0 (t) = sin t and
y 00 (t) = 2 cos t. By substitution, cos t is a solution if and only if 2 9 =
0. This equation has roots = 3
Problem 1.16
For what value(s) of m is y = emt a solution of the equation y 00 +3y 0 +2y = 0?
Solution.
2
Since dtd (emt ) = memt and dtd 2 (emt ) = m2 emt the requirement on m becomes
m2 + 3m + 2 = 0. Factoring the left-hand side to obtain (m + 2)(m + 1) = 0.
Thus, m = 2 or m = 1
Problem 1.17
Show that y(t) = et is a solution to the differential equation




2
2
00
0
y 2+
y + 1+
y = 0.
t
t
Solution.
Substituting y(t) = y 0 (t) = y 00 (t) = et into the equation we find








2
2
2 t
2 t
00
0
t
y + 1+
y =e 2 +
e + 1+
e
y 2+
t
t
t
t
2
2
=et 2et et + et + et = 0
t
t
Problem 1.18
Show that any function of the form y(t) = C1 cos t + C2 sin t satisfies the
differential equation
d2 y
+ 2 y = 0.
dt2

11
Solution.
Finding the first and the second derivatives of y we obtain
y 0 (t) = C1 sin t + C2 cos t
and
y 00 (t) = C1 2 cos t C2 2 sin t.
Substituting into the equation to obtain
d2 y
+ 2 y = C1 2 cos t C2 2 sin t + 2 (C1 cos t + C2 sin t)
dt2
=0
Problem 1.19
Suppose y(t) = 2e4t is the solution to the initial value problem y 0 + ky =
0, y(0) = y0 . Find the values of k and y0 .
Solution.
Since y(0) = 2, we have y0 = 2. The given function satisfies the equation
y 0 + ky = 0, that is, 8e4t + 2ke4t = 0. Dividing through by 2e4t > 0 to
obtain 4 + k = 0. Thus, k = 4
Problem 1.20
Consider t > 0. For what value(s) of the constant n, if any, is y(t) = tn a
solution to the differential equation
t2 y 00 2ty 0 + 2y = 0?
Solution.
Since t2 y 00 2ty 0 + 2y = t2 (n(n 1)tn2 ) 2t(ntn1 ) + 2tn = 0 we find
n(n 1) 2n + 2 = 0 or n2 3n + 2 = 0. This last equation can be factored
as (n 1)(n 2) = 0. Solving we find n = 1 or n = 2
Problem 1.21
(a) Show that y(t) = C1 e2t + C2 e2t is a solution of the differential equation
y 00 4y = 0, where C1 and C2 are arbitrary constants.
(b) Find the solution satisfying y(0) = 2 and y 0 (0) = 0.
(c) Find the solution satisfying y(0) = 2 and limt y(t) = 0

12

1 BASIC TERMINOLOGY

Solution.
(a) Finding the first and second derivatives of y(t) to obtain y 0 (t) = 2C1 e2t
2C2 e2t and y 00 (t) = 4C1 e2t + 4C2 e2t . Thus, y 00 4y = 4C1 e2t + 4C2 e2t
4(C1 e2t + C2 e2t ) = 0.
(b) The condition y(0) = 2 implies that C1 + C2 = 2. The condition y 0 (0) = 0
implies that 2C1 2C2 = 0 or C1 = C2 . But C1 + C2 = 2 and this implies
that C1 = C2 = 1. In this case, the particular solution is y(t) = e2t + e2t .
(c) The first condition implies that C1 +C2 = 2. The second condition implies
that C1 = 0 since limt et = . Thus, C2 = 2 and the particular solution
is given by y(t) = 2e2t
Problem 1.22
Suppose that the graph below is the particular solution to the initial value
problem y 0 (t) = m + 1, y(1) = y0 . Determine the constants m and y0 and
then find the formula for y(t).

Solution.
From the figure we see that y0 = y(1) = 1. Since y is the slope of the line
which is 1 we have y 0 (t) = 1 = m + 1. Solving for m we find m = 2.
Hence, y(t) = t + 2
Problem 1.23
Suppose that the graph below is the particular solution to the initial value
problem y 0 (t) = mt, y(t0 ) = 1. Determine the constants m and t0 and then
find the formula for y(t).

13

Solution.
From the graph we see that y(0) = 1 so that t0 = 0. Also, by integration
we see that y = m2 t2 + C. From the figure we see that C = 1. Finally, since
y(1) = 0.5 we find 12 = m2 1. Solving for m we find m = 1. Thus,
2
y(t) = t2 1
Problem 1.24
Show that y(t) = e2t is not a solution to the differential equation y 00 + 4y = 0.
Solution.
Finding the second derivative and substituting into the equation we find
y 00 + 4y = 4e2t + 4e2t = 8e2t 6= 0.
Thus, y(t) = e2t is not a solution to the given equation
Problem 1.25
Consider the initial-value problem
y 0 + 3y = 6t + 5, y(0) = 3.
(a) Show that y = Ce3t + 2t + 1 is a solution to the above differential
equation.
(b) Find the value of C.

14

1 BASIC TERMINOLOGY

Solution.
(a) Substituting y and y 0 into the equation we find
3Ce3t + 2 + 3[Ce3t + 2t + 1] = 3Ce3t + 3Ce3t + 6t + 5 = 6t + 5
(b) Since y(0) = 3 we find C + 1 = 3. Solving for C we find C = 2. Thus,
the solution to the initial value problem is y(t) = 2e3t + 2t + 1

2 Existence and Uniqueness of


Solutions to First Order Linear
IVP
Problem 2.1
2
Find p(t) and y0 so that the function y(t) = 3et is the solution to the IVP
y 0 + p(t)y = 0, y(0) = y0 .
Solution.
2
Since y(t) = 3et , we have y(0) = y0 = 3e0 = 3. On the other hand, y(t)
2
2
satisfies the equation y 0 + p(t)y = 0 so that 6tet + p(t)3et = 0. Solving for
p(t), we find p(t) = 2t
Problem 2.2
For each of the initial conditions, determine the largest interval a < t < b on
which Theorem 2.2 guarantees the existence of a unique solution.
1
y = sin t
y0 + 2
t +1
(a) y(0) = (b) y() = 0.
Solution.
Here we have p(t) =
(a) (, ).
(b) (, )

1
t2 +1

and g(t) = sin t.

Problem 2.3
For each of the initial conditions, determine the largest interval a < t < b on
which Theorem 2.2 guarantees the existence of a unique solution
y0 +

t
et
y
=
t2 4
t3
15

162 EXISTENCE AND UNIQUENESS OF SOLUTIONS TO FIRST ORDER LINEAR IVP


(a) y(5) = 2

(b) y( 32 ) = 1

(c) y(6) = 2.

Solution.
Notice that p(t) and g(t) are defined for all t 6= 2, 2, 3
(a) 3 < t < .
(b) 2 < t < 2.
(c) < t < 2
Problem 2.4
(a) For what values of the constant C and the exponent r is y = Ctr the
solution of the IVP
2ty 0 6y = 0, y(2) = 8?
(b) Determine the largest interval of the form a < t < b on which Theorem
2.2 guarantees the existence of a unique solution.
(c) What is the actual interval of existence for the solution found in part (a)?
Solution.
(a) Substitution leads to 2trCtr1 6Ctr = 0. Divide through by Ctr to
obtain 2r 6 = 0 or r = 3. Now, since y(2) = 8 we have C(2)3 = 8 or
C = 1. Thus, y(t) = t3 .
(b) Rewriting the equation in the form
3
y0 y = 0
t
so that p(t) = 3t and g(t) = 0. The largest interval of the form a < t < b
that guarantees the existence of a unique solution is the interval < t < 0
since 2 is in that interval and both p(t) and g(t) are continuous over that
interval.
(c) By part (a) the actual interval of existence is the set of all real numbers
since y(t) is a cubic polynomial
Problem 2.5
What information does Theorem 2.2 gives about the initial value problem
ty 0 = y + t3 cos t, y(1) = 1?y(1) = 1?
Solution.
We have p(t) = 1t and g(t) = t2 cos t. The interval of existence is 0 < t <
if y(1) = 1 and < t < 0 if y(1) = 1

17
Problem 2.6
Consider the following differential equation
(t 4)y 0 + 3y =

t2

1
.
+ 5t

Without solving, find the interval over which a unique solution is guaranteed
for each of the following initial conditions:
(a) y(3) = 4 (b) y(1.5) = 2 (c) y(6) = 0 (d) y(4.1) = 3.
Solution.
Rewriting the equation in the form
y0 +

3
1
y=
t4
(t 4)(t2 + 5t)

we find that p(t) and g(t) are continuous for all t 6= 5, 0, 4.


(a) 5 < t < 0.
(b) 0 < t < 4.
(c) < t < 5.
(d) 4 < t <
Problem 2.7
2
Without solving the initial value problem, (t1)y 0 +(ln t)y = t3
, y(t0 ) = y0 ,
state whether or not a unique solution is guaranteed to exist for the y(t0 ) = y0
listed below. If a unique solution is guaranteed, find the largest interval for
which the solution satisfies the differential equation and the initial condition.
(a) y(2) = 4 (b) y(0) = 0 (c) y(4) = 2.
Solution.
Rewriting the equation in the form
y0 +

ln t
2
y=
t1
(t 3)(t 1)

we find that p(t) and g(t) are continuous on (0, 1) (1, 3) (3, ).
(a) 1 < t < 3.
(b) No such solution.
(c) 3 < t <

182 EXISTENCE AND UNIQUENESS OF SOLUTIONS TO FIRST ORDER LINEAR IVP


Problem 2.8
(a) State precisely the theorem (hypothesis and conclusion) for existence and
uniqueness of a linear first order initial value problem.
3
(b) Consider the equation y 0 + t2 y = et with initial conditions y(t0 ) = y0 .
Briefly discuss if this has a solution, if it is unique and why.
Solution.
(a) If p(t) and g(t) are continuous functions in the open interval I = (a, b)
and t0 a point inside I then the IVP
y 0 + p(t)y = g(t), y(t0 ) = y0
has a unique solution y(t) defined on I.
3
(b) Since p(t) = t2 and g(t) = et are continuous in < t < , the IVP
has a unique solution for any t0
Problem 2.9
Is the differential equation linear or non-linear? If the equation is linear,
decide whether it is homogeneous or non-homogeneous.
(a) y 0 = ty 2 .
(b) y 0 = t2 y.
(c) (cos t)y 0 + et y = sin t.
0
(d) yy + t3 = sin t, y > 0.
Solution.
(a) Non-linear because of y 2 .
(b) Linear and homogeneous with p(t) = t2 and g(t) = 0.
et
(c) Linear and non-homogeneous with p(t) = cos
and g(t) = tan t.
t
(d) y 0 + (t3 sin t)y = 0. This is linear and homogeneous with p(t) = t3 sin t
and g(t) = 0
Problem 2.10
Consider the initial value problem
y 0 + p(t)y = g(t), y(3) = 1.
Suppose that p(t) and/or g(t) have discontinuities at t = 2, t = 0, and t = 5
but are continuous for all other values of t. What is the largest interval (a, b)
on which Theorem 2.2 is applied.

19
Solution.
Because of the initial condition the largest interval of existence guaranteed
by the existence and uniqueness theorem is 0 < t < 5
Problem 2.11
Determine and y0 so that the graph of the solution to the initial-value
problem
y 0 + y = 0, y(0) = y0
passes through the points (1, 4) and (3, 1).
Solution.
The general solution is given by y(t) = y(0)et . Since y(1) = 4 and y(3) = 1
we find
y(0)e
= 4.
y(0)e3
t
Solving for we find = ln24 = ln 2. Thus, y(t) = y(0)e(ln 2)t = y(0) 12 .
Since y(1) = 4 we have y(0)
= 4 so that y0 = 8
2
Problem 2.12
Match the following equations with the correct description. Every equation
matches exactly one description.
(a) y 0 = 3y 5t.
2
2y
(b) yt = t2y + x
2.
(c) y 0 y 2 = sin t.
(d) y 0 + 3y = 0.
(i) A partial differential equation.
(ii) A homogeneous first order linear differential equation.
(iii) A non-linear first order differential equation.
(iv) A non-homogenous first order linear differential equation.
Solution.
(a) (iv) (b) (i) (c) (iii) (d) (ii)
Problem 2.13
Consider the differential equation y 0 = t2 y + sin y. What is the order of this
equation? Is it linear or non-linear?

202 EXISTENCE AND UNIQUENESS OF SOLUTIONS TO FIRST ORDER LINEAR IVP


Solution.
A non-linear ( because of sin y) first order ordinary differential equation
Problem 2.14
Show that y 0 =
Solution.
We have y 0 =

t+y
t

t+y
t

is a linear first order non-homogeneous equation.

= 1 + yt . Thus,
1
y 0 y = 1.
t

This is a first-order linear non-homogeneous equation with p(t) = 1t and


g(t) = 1

3 Analytical Solution: The


Method of Integrating Factor
Problem 3.1
Solve the IVP
y 0 = 2ty, y(1) = 1.
Solution.
First we rearrange the equation to the form recognizable as first-order linear:
y 0 + 2ty = 0.
R
From this we see that p(t) = 2t so that 2tdt = t2 . Thus, the general solution
2
2
to the DE is y(t) = Cet . But y(1) = 1 so that C = e. Hence, y(t) = e1t
Problem 3.2
Solve the IVP
y 0 + et y = 0, y(0) = 2.
Solution.
R
Since p(t) = et we find et dt = et so that the general solution to the DE is
t
y(t) = Cee . But y(0) = 2 so that C = 2e. Hence, the unique solution is
t
y(t) = 2e1e
Problem 3.3
Consider the first order linear non-homogeneous IVP
y 0 + p(t)y = p(t), y(t0 ) = y0 .
(a) Show that the IVP can be reduced to a first order linear homogeneous
IVP by the change of variable z(t) = y(t) .
(b) Solve this initial value problem for z(t) and use the solution to determine
y(t).
21

223 ANALYTICAL SOLUTION: THE METHOD OF INTEGRATING FACTOR


Solution.
(a) Note that the given DE can be written as y 0 + p(t)(y ) = 0. Since
z(t) = y(t) , we get the IVP
z 0 + p(t)z = 0, z(t0 ) = y(t0 ) .
(b)The general
solution to the DE is z(t) = (y0 )e
R
tt p(s)ds
(y0 )e 0
+

Rt
t0

p(s)ds

. Thus, y(t) =

Problem 3.4
Apply the results of the previous problem to solve the IVP
y 0 + 2ty = 6t, y(0) = 4.
Solution.
Letting z(t) = y(t) 3, the given IVP reduces to
z 0 + 2tz = 0, z(0) = 1.
2

The unique solution to this IVP is z(t) = et . Hence, y(t) = et + 3


Problem 3.5
The unique solution to the IVP
ty 0 y = 0, y(1) = y0
goes through the points (2, 1) and (4, 4). Find the values of and y0 .
Solution.
Rewriting the given IVP in the standard form
y0

y = 0, y(1) = y0 ,
t

R
we find p(t) = t and t dt = ln |t| = ln |t |. Thus, the general

solution to the DE is given by y(t) = Celn |t | = C|t| . But y(2) = 1 and


y(4) = 4 so that C2 = 1 and C4 = 4. Taking the ratio of these last
equations we find 2 = 4 and thus = 2. From C2 = 1 we find 4C = 1
or C = 0.25. Hence, y0 = y(1) = 0.25(1)2 = 0.25

23
Problem 3.6
The table below lists values of t and ln [y(t)] where y(t) is the unique solution
to the IVP
y 0 + tn y = 0, y(0) = y0 .
t
1
2
3
4
ln [y(t)] -0.25 -4.00 -20.25 -64.00
(a) Determine the values of n and y0 .
(b) What is y(1)?
Solution.
tn+1
(a) The general solution to the DE is y(t) = Ce n+1 . Since y(0) = y0 we
tn+1

find C = y0 so that the unique solution is y(t) = y0 e n+1 . Thus, ln [y(t)] =


n+1
1
ln (y0 ) tn+1 . Since ln y(1) = 14 and ln y(2) = 4 we find ln y0 n+1

1
15
2n+1 1
15
2n+1
ln y0 + n+1 = 4 4 = 4 . Thus, n+1 = 4 . Using trial and error on the
1
= 14 + 41 = 0 so that
values of n, one finds n = 3. Hence, ln y0 = 14 + n+1
y0 = 1.

(1)4
4
(b) y(1) = 1 e 4 = e1
Problem 3.7
Consider the differential equation y 0 + p(t)y = 0. Find p(t) so that y =
the general solution.

c
t

is

Solution.
Substituting in the equation we find
c
c
2 + p(t) = 0.
t
t
Solving for p(t) we find p(t) =

1
t

Problem 3.8
Consider the differential equation y 0 + p(t)y = 0. Find p(t) so that y = ct3 is
the general solution.
Solution.
Substituting in the equation we find
3ct2 + p(t)(ct3 ) = 0.
Solving for p(t) we find p(t) = 3t

243 ANALYTICAL SOLUTION: THE METHOD OF INTEGRATING FACTOR


Problem 3.9
Solve the initial-value problem: y 0 3t y = 0, y(2) = 8.
Solution.
Using the method of integrating factor, we find y(t) = Ct3 . From y(2) = 8,
we find C23 = 8 and thus C = 1. The unique solution to the initial-value
problem is y(t) = t3
Problem 3.10
Solve the differential equation y 0 2ty = 0.
Solution.
Since
p(t) = 2t, using the method of integrating factor, we find y(t) =
R
2
2tdt
Ce
= Cet
Problem 3.11
Find the function f (t) that crosses the point (0, 4) and whose slope satisfies
f 0 (t) = 2f (t).
Solution.
Solving the differential equation we find f (t) = Ce2t . Since f (0) = 4, we find
C = 4. Thus, f (t) = 4e2t
Problem 3.12
Find the general solution to the differential equation y 00 2y 0 = 0. Hint: Let
y 0 = z.
Solution.
Let z = y 0 so that z 0 = y 00 . Thus, z 0 2z = 0. Solving this DE by the method
of integrating factor, we find z(t) = Ce2t = y 0 . Hence, y(t) = Ce2t + C 0
Problem 3.13
Solve the IVP: y 0 + 2ty = t, y(0) = 0.
Solution.
R
2
Since p(t) = 2t, we find (t) = e 2tdt = et . Multiplying the given equation
2
by et to obtain
 2 0
2
et y = tet .

25
Integrating both sides with respect to t and using substitution on the righthand integral to obtain
1 2
2
et y = et + C.
2
t2
Dividing the last equation by e to obtain
1
2
y(t) = Cet + .
2
Since y(0) = 0, we find C = 12 . Thus, the unique solution to the IVP is
given by
1
2
y = (1 et )
2
Problem 3.14
Find the general solution: y 0 + 3y = t + e2t .
Solution.
Since p(t) = 3, the integrating factor is (t) = e3t . Thus, the general solution
is
Z
3t
y(t) =e
e3t (t + e2t )dt + Ce3t
Z
3t
=e
(te3t + et )dt + Ce3t

 3t
e
t
3t
(3t 1) + e + Ce3t
=e
9
3t 1
+ e2t + Ce3t
=
9
Problem 3.15
Find the general solution: y 0 + 1t y = 3 cos t, t > 0.
Solution.
R dt
Since p(t) = 1t , the integrating factor is (t) = e t = eln t = t. Using the
method of integrating factor we find
Z
1
C
y(t) =
3t cos tdt +
t
t
C
3
= (t sin t + cos t) +
t
t
3 cos t C
=3 sin t +
+
t
t

263 ANALYTICAL SOLUTION: THE METHOD OF INTEGRATING FACTOR


Problem 3.16
Find the general solution: y 0 + 2y = cos (3t).
Solution.
We have p(t) = 2 so that (t) = e2t . Thus,
Z
2t
y(t) = e
e2t cos (3t)dt + Ce2t
But
Z
e2t
2
e cos (3t)dt = sin (3t)
e2t sin (3t)dt
3
3

 2t
Z
2
e
2
e2t
2t
cos (3t) +
e cos (3t)dt
= sin (3t)
3
3
3
3
Z
13
e2t
2t
e cos (3t)dt = (3 sin (3t) + 2 cos (3t))
9
9
Z
e2t
2t
e cos (3t)dt = (3 sin (3t) + 2 cos (3t)).
13
Z

2t

Hence,
y(t) =

1
(3 sin (3t) + 2 cos (3t)) + Ce2t
13

Problem 3.17
Given that the solution to the IVP ty 0 + 4y = t2 , y(1) = 13 exists on the
interval < t < . What is the value of the constant ?
Solution.
Solving this equation with the integrating factor method with p(t) =
find (t) = t4 . Thus,
Z
1
C
y(t) = 4 t4 (t)dt + 4
t
t

C
= t2 + 4 .
6
t

4
t

we

Since the solution is assumed to be defined for all t, we must have C = 0.


Since y(1) = 13 we find = 2

27
Problem 3.18
Suppose that y(t) = Ce2t + t + 1 is the general solution to the equation
y 0 + p(t)y = g(t). Determine the functions p(t) and g(t).
Solution.
R
The integrating factor is (t) = e2t . Thus, p(t)dt = 2t and this implies that
p(t) = 2. On the other hand, the function t + 1 is the particular solution
to the non-homogeneous equation so that (t + 1)0 + 2(t + 1) = g(t). Hence,
g(t) = 2t + 3
Problem 3.19
Suppose that y(t) = 2et + et + sin t is the unique solution to the IVP
y 0 + y = g(t), y(0) = y0 . Determine the constant y0 and the function g(t).
Solution.
First, we find y0 : y0 = y(0) = 2 + 1 + 0 = 1. Next, we find g(t) : g(t) =
y 0 + y = (2et + et + sin t)0 + (2et + et + sin t) = 2et + et + cos t 2et +
et + sin t = 2et + cos t + sin t
Problem 3.20
Find the value (if any) of the unique solution to the IVP y 0 + (1 + cos t)y =
1 + cos t, y(0) = 3 in the long run, i.e., limr y(t)?
Solution.
R
The integrating factor is (t) = e (1+cos t)dt = et+sin t . Thus, the general solution is
Z
(t+sin t)
y(t) =e
et+sin t (1 + cos t)dt + Ce(t+sin t)
=1 + Ce(t+sin t)
Since y(0) = 3 we find C = 2 and therefore y(t) = 1 + 2e(t+sin t) . Finally,
lim y(t) = lim (1 + 2e sin t et ) = 1

Problem 3.21
Find the solution to the IVP
y 0 + p(t)y = 2, y(0) = 1
where


p(t) =

0, if 0 t 1
1
if 1 < t 2.
t

283 ANALYTICAL SOLUTION: THE METHOD OF INTEGRATING FACTOR


Solution.
First, we solve the IVP
y 0 = 2, y(0) = 1, 0 t 1.
The general solution is y1 (t) = 2t + C. Since y(0) = 1, we find C = 1. Hence,
y1 (t) = 2t + 1 and y(1) = 3.
Next, we solve the IVP
1
y 0 + y = 2, y(1) = 3, 1 < t 2.
t
The integrating factor is (t) = t and the general solution is y2 (t) = t + Ct .
Since y2 (1) = 3, we find C = 2. Thus,

2t + 1, if 0 t 1
y(t) =
t + 2t if 1 < t 2
Problem 3.22
Find the solution to the IVP
y 0 + (sin t)y = g(t), y(0) = 3
where


g(t) =

sin t,
if 0 t
sin t if < t 2.

Solution.
First, we solve the IVP
y 0 + (sin t)y = sin t, y(0) = 3, 0 t
The integrating factor is (t) = e cos t and the general solution is y1 (t) =
1 + Cecos t . Since y1 (0) = 3 we find C = 2e1 . Hence, y1 (t) = 1 + 2ecos t1 and
y1 () = 1 + 2e2 .
Next, we solve the IVP
y 0 + sin ty = sin t, y() = 1 + 2e2 , < t 2
The integrating factor is (t) = e cos t and the general solution
is y2 (t) =

1 + Cecos t . Since y2 () = 1 + 2e2 we find C = 2 1e + e . Thus,

1 + 2ecos t1
if 0 t
 cos t
y(t) =
1
1 + 2 e + e e
if < t 2

29
Problem 3.23
Find the solution to the IVP
y 0 + y = g(t), t > 0, y(0) = 3
where

g(t) =

1, if 0 t 1
0
if t > 1.

Solution.
First, we solve the IVP
y 0 + y = 1, y(0) = 3, 0 t 1.
The integrating factor is (t) = et and the general solution is y1 (t) = 1+Cet .
Since y1 (0) = 3, we find C = 2. Hence, y1 (t) = 1 + 2et and y1 (1) = 1 + 2e1 .
Next, we solve the IVP
y 0 + y = 0, y(1) = 1 + 2e1 , t > 1.
The integrating factor is (t) = et and the general solution is y2 (t) = Cet
Since y2 (1) = 1 + 2e1 , we find C = 2 + e. Thus,

y(t) =

1 + 2et , if 0 t 1
(2 + e)et if t > 1

Problem 3.24
Find the solution to the IVP
y 0 + p(t)y = 0, y(0) = 3
where

2t 1, if 0 t 1
0
if 1 < t 3
p(t) =

1t
if 3 < t 4.
Sketch an accurate graph of the solution, and discuss the long-term behavior
of the solution. Is the solution differentiable on the interval t > 0? Explain
your answer.

303 ANALYTICAL SOLUTION: THE METHOD OF INTEGRATING FACTOR


Solution.
First, we solve the IVP
y 0 + (2t 1)y = 0, y(0) = 3, 0 t 1.
2

The integrating factor is (t) = et t and the general solution is y1 (t) =


2
2
Ce(t t) . Since y1 (0) = 3, we find C = 3. Hence, y1 (t) = 3e(t t) and
y1 (1) = 3.
Next, we solve the IVP
y 0 = 0, y(1) = 3, 1 < t 3.
The general solution is y2 (t) = C. Since y2 (1) = 3, we find C = 3 and
y2 (t) 3.
Next, we solve the IVP
1
y 0 y = 0, y(3) = 3, 3 < t 4.
t
The integrating factor is (t) = 1t and the general solution is y3 (t) = Ct.
Since y3 (3) = 3, we find C = 1. Hence, y3 (t) = t. Hence,

2
3e(t t) , if 0 t 1
y(t) =
3,
if 1 < t 3

t
if 3 < t 4.
The graph of y(t) is shown below

It follows that limt y(t) = . The function y(t) is not differentiable at


t = 1 and t = 3 on the domain t > 0
Problem 3.25
Solve the initial-value problem y 0 + y = et y 2 , y(0) = 1 using the substitution
1
u(t) = y(t)
.

31
Solution.
Substituting into the equation we find
u0 u = et , u(0) = 1.
Solving this equation by the method of integrating factor with (t) = et ,
we find
u(t) = tet + Cet .
Since u(0) = 1, we find C = 1 and therefore u(t) = tet + et . The general
solution is
y(t) = (tet + et )1

323 ANALYTICAL SOLUTION: THE METHOD OF INTEGRATING FACTOR

4 Existence and Uniqueness of


Solutions to First Order
Nonlinear IVP
For the given initial value problem in Problems 4.1 - 4.5,
(a) Rewrite the differential equation, if necessary, to obtain the form
y 0 = f (t, y), y(t0 ) = y0 .
Identify the function f (t, y).
. Determine where in the typlane both f (t, y) and f
are
(b) Compute f
y
y
continuous.
(c) Determine the largest open rectangle in the typlane that contains the
point (t0 , y0 ) and in which the hypotheses of Theorem 4.1 are satisfied.
Problem 4.1

3y 0 + 2t cos y = 1, y( ) = 1.
2
Solution.
(a) y 0 = 13 (1 2t cos y) = f (t, y).
(b) f
(t, y) = 32 t sin y. The functions f (t, y) and fy (t, y) are both continuous
y
in the entire plane,
D = {(t, y) : < t < , < y < }.
(c) R = {(t, y) : < t < , < y < }
33

344 EXISTENCE AND UNIQUENESS OF SOLUTIONS TO FIRST ORDER NONLINEAR IV


Problem 4.2

3ty 0 + 2 cos y = 1, y( ) = 1.
2
Solution.
(a) y 0 = 3t1 (1 2 cos y) = f (t, y).
(b) fy (t, y) = 3t2 sin y. Both f (t, y) and fy (t, y) are continuous in
D = {(t, y) : < t < 0, 0 < t < , < y < }.
(c) R = {(t, y) : 0 < t < , < y < }
Problem 4.3
2t + (1 + y 3 )y 0 = 0, y(1) = 1.
Solution.
2t
(a) y 0 = 1+y
3 = f (t, y).
(b) fy (t, y) =

6ty 2
.
(1+y 3 )2

Both f (t, y) and fy (t, y) are continuous in

D = {(t, y) : < t < , < y < 1, 1 < y < }.


(c) R = {(t, y) : < t < , 1 < y < }
Problem 4.4
(y 2 9)y 0 + ey = t2 , y(2) = 2.
Solution.
2
y
(a) y 0 = t ye
2 9 = f (t, y).
(b) fy (t, y) =

(y 2 +2y9)ey 2t2 y 2
.
y 2 9

Both f (t, y) and fy (t, y) are continuous in

D = {(t, y) : < t < , < y < 3, 3 < y < 3, 3 < y < }.


(c) D = {(t, y) : < t < , 3 < y < 3}
Problem 4.5
cos yy 0 = 2 + tan t, y(0) = 0.

35
Solution.
t
= f (t, y).
(a) y 0 = 2+tan
cos y
(b) fy (t, y) = (2 + tan t) sec y tan y. Both f (t, y) and fy (t, y) are continuous
in
D = {(t, y) : t 6= (2n + 1) 2 , y 6= (2m + 1) 2 , where n and m are integers}.
(c) R = {(t, y) : 2 < t < 2 ,

< y < 2 }

Problem 4.6
Give an example of an initial value problem for which the open rectangle
R = {(t, y) : 0 < t < 4, 1 < y < 2}
represents the largest region in the typlane where the hypotheses of Theorem 4.1 are satisfied.
Solution.
An example is
y0 =

1
, y(2) = 0
t(t 4)(y + 1)(y 2)

Problem 4.7
Can we apply Theorem 4.1 to the following problem ? Explain what (if
anything) we can conclude, and why (or why not):
y
y 0 = , y(0) = 2.
t
Solution.
Since f (t, y) =
region

and fy (t, y) =

,
t

both functions are continuous in the

D = {(t, y) : 0 < t < , < y < }.


Since (0, 2) is not in D, Theorem 4.1 can not be applied in this case
Problem 4.8
Consider the differential equation y 0 = ty
. For which of the following initial
t+y
value conditions does Theorem 4.1 apply?
(a) y(0) = 0 (b) y(1) = 1 (c) y(1) = 1.

364 EXISTENCE AND UNIQUENESS OF SOLUTIONS TO FIRST ORDER NONLINEAR IV


Solution.
is continuous everywhere except along the line
The function f (t, y) = ty
t+y
t + y = 0. (a) and (b): Since both (0, 0) and (1, 1) lie on this line, we
cannot conclude existence from Theorem 4.1.
(c) The point (1, 1) is not on that line so we can find a small rectangle
around this point where Theorem 4.1 guarantees the existence of a solution.
2t
Furthermore, since fy (t, y) = (t+y)
2 is continuous at (1, 1), the solution
is unique
Problem 4.9
Does the initial value problem y 0 =
Theorem 4.1?

y
t

+ 2, y(0) = 1 satisfy the conditions of

Solution.
The equation is of the form y 0 = f (t, y) = yt + 2. The function f is continuous
outside the line t = 0. The initial value point is (0,1), so there is no rectangle
containing it in which f is continuous, and the conditions of Theorem 4.1 are
not satisfied
Problem 4.10
Does the initial value problem y 0 = y sin y + t, y(0) = 1 satisfy the conditions of Theorem 4.1?
Solution.
The equation is of the form y 0 = f (t, y) = y sin y + t. the function f is
continuous in the whole plane, and so is its partial derivative fy (t, y) =
sin y + y cos y. In particular, any rectangle around the initial value point will
satisfy the conditions of Theorem 4.1

5 Separable Differential
Equations
Problem 5.1
Solve the (separable) differential equation
2 ln y 2

y 0 = tet

Solution.
At first, this equation may not appear separable, so we must simplify the
right hand side until it is clear what to do.
2 ln y 2

y 0 =tet

t2

ln

=te e
1
2
=tet 2
y
t t2
= 2e .
y

1
y2

Separating the variables and solving the equation we find


2

y 2 y 0 =tet
Z
Z
1
2
3 0
(y ) dt = tet dt
3
1 3 1 t2
y = e +C
3
2
3
2
y 3 = et + C
2
37

38

5 SEPARABLE DIFFERENTIAL EQUATIONS

Problem 5.2
Solve the (separable) differential equation
y0 =

t2 y 4y
, t 6= 2.
t+2

Solution.
Separating the variables and solving we find
y 0 t2 4
=
=t2
y
t+2
Z
Z
0
(ln |y|) dt = (t 2)dt
ln |y| =

t2
2t + C
2
t2

y(t) =Ce 2 2t
Problem 5.3
Solve the (separable) differential equation
ty 0 = 2(y 4).
Solution.
Separating the variables and solving we find
2
y0
=
y4 t
Z
Z
2
0
(ln |y 4|) dt =
dt
t
ln |y 4| = ln t2 + C
y4
ln | 2 | =C
t
y(t) =Ct2 + 4
Problem 5.4
Solve the (separable) differential equation
y 0 = 2y(2 y).

39
Solution.
Separating the variables and solving (using partial fractions in the process)
we find
y0
=2
y(2 y)
y0
y0
+
=2
2y 2(2 y)
y0
y0
+
=4
y
(2 y)
Z
Z
Z
1
0
0
(ln |y|) dt
(ln |2 y|) dt = 4dt
2


y
=4t + C

ln
2 y


y
4t


2 y =Ce
2Ce4t
y(t) =
.
1 + Ce4t
Note that y(t) 2 is a singular solution
Problem 5.5
Solve the IVP:
y0 =

4 sin (2t)
, y(0) = 1.
y

Solution.
Separating the variables and solving we find
yy 0 =4 sin (2t)
(y 2 )0 =8 sin (2t)
Z
Z
2 0
(y ) dt = 8 sin (2t)dt
y 2 = 4 cos (2t) + C
p
y(t) = C 4 cos (2t).
Since y(0) = 1, we find C = 5 and hence
p
y(t) = 5 4 cos (2t)

40
Problem 5.6
Solve the IVP:

5 SEPARABLE DIFFERENTIAL EQUATIONS

yy 0 = sin t, y( ) = 2.
2

Solution.
Separating the variables and solving we find
Z 

y2
2

0

Z
dt =

sin tdt

y2
= cos t + C
2
y 2 = 2 cos t + C.
Since y( 2 ) = 2, we find C = 4. Thus, y(t) =
p
y( 2 ) = 2, y(t) = (2 cos t + 4)

p
(2 cos t + 4). Since

Problem 5.7
Solve the IVP:
y0 +

1
= 0, y(1) = 0.
y+1

Solution.
Separating the variables and solving we find
d
[(1 + y)2 ] = 2
dt
(y + 1)2 = 2t + C

y + 1 = 2t + C

y(t) = 2t + C 1.
Since y(1) = 0, we find C = 3. Thus, y(t) =

2t + 3 1

Problem 5.8
Solve the IVP:
y 0 ty 3 = 0, y(0) = 2.

41
Solution.
Separating the variables and solving we find
Z

0 3

y y dt = tdt
Z  2 0
t2
y
dt = + C
2
2
1
2 =t2 + C
y
1
.
y2 = 2
t + C
q
Since y(0) = 2, we find C = 14 . Thus, y(t) = 4t42 +1 . Since y(0) = 2, we
have y(t) =

2
4t2 +1

Problem 5.9
Solve the IVP:

y 0 = 1 + y 2 , y( ) = 1.
4
Solution.
Separating the variables and solving we find
y0
=1
1 + y2
arctan y =t + C
y(t) = tan (t + C).
Since y( 4 ) = 1, we have C = 2 . Hence, y(t) = tan t
Problem 5.10
Solve the IVP:
1
y 0 = t ty 2 , y(0) = .
2

42

5 SEPARABLE DIFFERENTIAL EQUATIONS

Solution.
Separating the variables and solving we find
y0
=t
y2 1
y0
y0

= 2t
y1 y+1


y 1
= 2t + C
ln
y + 1
y1
=Ce2t
y+1
1 + Ce2t
y(t) =
.
1 Ce2t
Since y(0) = 12 , we find C = 13 . Thus,
3 e2t
y(t) =
3 + e2t
Problem 5.11
Solve the IVP:
(2y sin y)y 0 = sin t t, y(0) = 0.
Solution.
Separating the variables and solving we find
Z
Z
0
(2y sin y)y dt = (sin t t)dt
y 2 + cos y = cos t

t2
+ C.
2

Since y(0) = 0, we find C = 2. Thus,


y 2 + cos y + cos t +

t2
=2
2

Problem 5.12
For what values of the constants , y0 , and integer n is the function y(t) =
1
(4 + t) 2 a solution of the initial value problem
y 0 + y n = 0, y(0) = y0 ?

43
Solution.
3
1
We have y0 = y(0) = (4 + 0) 2 = 12 . Also, y 0 = 12 (4 + t) 2 = 21 y 3 . Thus,
1
y0 + y3 = 0
2
so that =

1
2

and n = 3

Problem 5.13
State an initial value problem, with initial condition imposed at t0 = 2,
having implicit solution y 3 + t2 + sin y = 4.
Solution.
Differentiating both sides of the given equation we find
3y 2 y 0 + (cos y)y 0 + 2t = 0, y(2) = 0
Problem 5.14
Consider the initial value problem
y 0 = 2y 2 , y(0) = y0 .
For what value(s) of y0 will the solution have a vertical asymptote at t = 4,
where the tinterval of existence is < t < 4?
Solution.
Solving the differential equation by the method of separating the variables
we find
y0
=2
y2
Z 0
Z
y
dt = 2dt
y2
1
=2t + C
y
1
y(t) =
.
C 2t
y0
Since y(0) = y0 , we find C = y10 . Thus, y(t) = 12y
. This function will have
0t
a vertical asymptote at t = 4 when 1 2(4)y0 = 0 or y0 = 81

44

5 SEPARABLE DIFFERENTIAL EQUATIONS

Problem 5.15
Assume that y sin y 3t + 3 = 0 is an implicit solution of the initial value
problem y 0 = f (y), y(1) = 0. What is f (y)? What is an implicit solution to
the initial value problem y 0 = t2 f (y), y(1) = 0?
Solution.
Taking the derivative of the given equation with respect to t we find
y 0 sin y + yy 0 cos y 3 = 0.
Thus,
y0 =

3
= f (y).
sin y + y cos y

If y 0 = t2 f (y) then
3t2
.
sin y + y cos y
Solving this equation by the method of separation of variables we find
y0 =

y 0 sin y + yy 0 cos y =3t2


(y sin y)0 =3t2
y sin y =t3 + C.
Since y(1) = 0, we find C = 1. Hence, the implicit solution is given by
y sin y t3 + 1 = 0
Problem 5.16
Find all the solutions to the differential equation y 0 =

2ty
.
1+t

Solution.
Separating the variables to obtain
y0
2t
2
=
=2
y 1+t
t+1
ln |y| =2t ln (t + 1)2 + C
ln |(t + 1)2 y| =2t + C
(t + 1)2 y =Ce2t
Ce2t
y(t) =
(t + 1)2

45
Problem 5.17
Solve the initial-value problem y 0 = cos2 y cos2 t, y(0) = 4 .
Solution.
Solving by the method of separation of variables we find
y0
1 + cos (2t)
= cos2 t =
2
cos y
2
t 1
tan y = + sin 2t + C.
2 4
Since y(0) = 4 , we find C = 1. Hence,
tan y =

t 1
+ sin 2t + 1
2 4

Problem 5.18
Solve the initial-value problem y 0 = et+y , y(0) = 0 and determine the interval
on which the solution y(t) is defined.
Solution.
Separating the variables we obtain
y 0 ey = et .
Integrating both sides to obtain
ey = et + C.
But y(0) = 0 so that C = 2. Hence, ey = et + 2. Solving for y we find
y(t) = ln(2 et ).
This function is defined for 2 et > 0, that is, for t < ln 2
Problem 5.19
(a) Solve the initial-value problem
y0 =

t2
ey

.
ey
t2

State the name of the method you are using.


(b) Find the solution which satisfies the condition y(1) = 1.

46

5 SEPARABLE DIFFERENTIAL EQUATIONS

Solution.
(a) Using the method of separation of variables we find
y 0 ey =t2
ey =

1
t2

t3 1
+ C.
3
t

(b) Since y(1) = 1, we find C = e1 + 34 . Thus, the unique solution is defined


implicitly by the expression
ey +

t3 1
4
+ = e1 +
3
t
3

6 Exact Differential Equations


Problem 6.1
Find f
and f
if f (t, y) = y ln y ety .
t
y
Solution.
f
t
f
y

= yety .

= ln y + 1 + tety

Problem 6.2
and f
if f (t, y) = ln (ty) +
Find f
t
y

t2 +1
.
y5

Solution.
f
t
f
y

1
t

1
y

2t
.
y5

t2 +1
(y5)2

Problem 6.3
Let f (u, v) = 2u 3uv where u(t) = 2 cos t and v(t) = 2 sin t. Find

df
.
dt

Solution.
By the Chain Rule
df f du f dv
=
+
dt u dt
v dt
=(2 3v)(2 sin t) 3u(2 cos t) = (2 6 sin t)(2 sin t) 6 cos t(2 cos t)
=12 sin2 t 12 cos2 t 4 sin t = 12 sin2 t 12(1 sin2 t) 4 sin t
=24 sin2 t 4 sin t 12
47

48

6 EXACT DIFFERENTIAL EQUATIONS

In Problems 6.4 - 6.8, determine whether the given differential equation is


exact. If the equation is exact, find an implicit solution and (where possible)
an explicit solution.
Problem 6.4
yy 0 + 3t2 2 = 0, y(1) = 2.
Solution.
We have M (t, y) = 3t2 2 and N (t, y) = y. Thus, M
(t, y) = 0 = N
(t, y) so
y
t
that the equation is exact.
Z
H
2
(t, y) = 3t 2 = H(t, y) = (3t2 2)dt = t3 2t + h(y).
t
But

H
(t, y)
y

= y so that h0 (y) = y and hence h(y) =


t3 2t +

y2
2

+ C. Therefore

y2
= C.
2

Since y(1) = 2, we find C = 3. It follows


t3 2t +

y2
= 3.
2

3
Solving for
y we find y(t) = 4t 2t + 6. Since y(1) = 2, we find
y(t) = 4t t3 + 6
Problem 6.5
y 0 = (3t2 + 1)(y 2 + 1), y(0) = 1.
Solution.
Since the equation is separable, it is exact. Integrating H
(t, y) = 3t2 + 1
t
H
3
with respect to t we find H(t, y) = t + t + h(y). But y (t, y) = (y 2 + 1)1
which implies that h0 (y) = (y 2 + 1)1 . Thus, h(y) = arctan y + C. Hence,
t3 + t arctan y = C.
Since y(0) = 1 we find C = 4 . It follows

t3 + t arctan y = .
4

49
Solving for y(t) we find


y(t) = tan t3 + t +
4
Problem 6.6
(6t + y 3 )y 0 + 3t2 y = 0, y(1) = 2.
Solution.
We have M (t, y) = 3t2 y and N (t, y) = 6t + y 3 . Since
N
(t, y) = 6, the given differential equation is not exact
t

M
(t, y)
y

= 3t2 and

Problem 6.7
(et+y + 2y)y 0 + (et+y + 3t2 ) = 0, y(0) = 0.
Solution.
We have M (t, y) = et+y +3t2 and N (t, y) = et+y +2y. Since
N
(t, y), the given differential equation is exact.
t
H
(t, y) = et+y + 3t2 = H(t, y) =
t

M
(t, y)
y

= et+y =

(et+y + 3t2 )dt = et+y + t3 + h(y).

Also
H
(t, y) = et+y + 2y = h0 (y) + et+y = h0 (y) = 2y = h(y) = y 2 + C.
y
Hence,
et+y + t3 + y 2 = C.
Since y(0) = 0 we find C = 1. Therefore,
et+y + t3 + y 2 = 1
Problem 6.8
(sin (t + y) + y cos (t + y) + t + y)y 0 + (y cos (t + y) + y + t) = 0, y(1) = 1.

50

6 EXACT DIFFERENTIAL EQUATIONS

Solution.
We have M (t, y) = y cos (t + y)+t+y and N (t, y) = sin (t + y)+y cos (t + y)+
t+y. Since M
(t, y) = cos (t + y)y sin (t + y)+1 = N
(t, y), the differential
y
t
equation is exact. Thus,
Z
H
(t, y) =y cos (t + y) + t + y = H(t, y) = (y cos (t + y) + t + y)dt
t
t2
=y sin (t + y) + + yt + h(y).
2
Also,
H
(t, y) = sin (t + y) + y cos (t + y) + t + y = y cos (t + y) + sin (t + y) + t + h0 (y)
y
y2
h0 (y) =y = h(y) =
+ C.
2
Hence,
t2
y2
+ ty +
= C.
2
2
Since y(1) = 1 we find C = 0. Therefore,
y sin (t + y) +

y sin (t + y) +

t2
y2
+ ty +
=0
2
2

Problem 6.9
For what values of the constants m, n, and (if any) is the following differential equation exact?
tm y 2 y 0 + t3 y n = 0.
Solution.
We have M (t, y) = t3 y n and N (t, y) = tm y 2 . Thus, M
(t, y) = nt3 y n1
y
and N
(t, y) = mtm1 y 2 . For the differential equation to be exact we must
t
M
have y (t, y) = N
(t, y), i.e.,
t
nt3 y n1 = mtm1 y 2 .
This shows that m 1 = 3 so that m = 4. Also, n 1 = 2 so that n = 3.
Finally, 3 = 4 so that = 34

51
Problem 6.10
Assume that N (t, y)y 0 + t2 + y 2 sin t = 0 is an exact differential equation.
Determine the general form of N (t, y).
Solution.
We have M (t, y) = t2 + y 2 sin t. Since the differential equation is exact,
N
(t, y) = M
(t, y) = 2y sin t. Hence,
t
y
Z
N (t, y) = 2y sin tdt = 2y cos t + h(y)
Problem 6.11
Assume that t3 y + et + y 2 = 5 is an implicit solution to the differential
equation
N (t, y)y 0 + M (t, y) = 0, y(0) = y0 .
Determine possible functions M (t, y), N (t, y), and the possible value(s) for
y0 .
Solution.
Replacing y by y0 and t by 0 to obtain y0 = 2. Differentiating the given
equation with respect to t we find 3t2 y + et + (t3 + 2y)y 0 = 0. Thus, M (t, y) =
3t2 + et and N (t, y) = t3 + 2y
Problem 6.12

Assume that y = t 4 t2 is an explicit solution of the following initial


value problem
(y + at)y 0 + (ay + bt) = 0, y(0) = y0 .
Determine values for the constants a, b and y0 .
Solution.

(t, y) =
We have y0 = 0 4 02 = 2. Since N
t
differential equation is exact. From this we have

M
(t, y)
y

= a, the

H
y2
(t, y) = y + at = H(t, y) =
+ aty + h(t)
y
2
and
H
b
(t, y) = ay + bt = ay + h0 (t) = h0 (t) = bt = h(t) = t2 + C.
t
2

52

6 EXACT DIFFERENTIAL EQUATIONS

Hence,
b
y2
+ aty + t2 = C.
2
2
Since y(0) = 2 we find C = 2. Therefore,
y 2 + 2aty + bt2 = 4.
Solving this quadratic equation for y we find
p
2at 4a2 t2 4(bt2 4)
y=
.
2
Thus,
p
y(t) = at a2 t2 (bt2 4).

Since y(0) = 2 we obtain y(t) = at t2 a2 bt2 + 4. Finally, a = 1, a2


b = 1, b = 2
Problem 6.13
Let k be a positive constant. Use the exactness criterion to determine
= kP is exact. Do NOT try
whether or not the population equation dP
dt
to solve the equation or carry out any further calculation.
Solution.
= 0 we find that M (t, P ) = kP
Rewriting the equation in the form kP dP
dt
N
M
and N (t, P ) = 1. Since t (t, P ) = P (t, P ) = 0, the differential equation
is exact
Problem 6.14
Consider the differential equation (2t+3)+(2y2)y 0 = 0. Determine whether
this equation is exact or not. If it is, solve it.
Solution.
We have M (t, y) = 2t+3 and N (t, y) = 2y 2. Since M
(t, y) = N
(t, y) = 0,
y
t
the differential equation is exact. Now,
Z
H
(t, y) = 2t + 3 = H(t, y) = (2t + 3)dt = t2 + 3t + h(y).
t
Also
H
(t, y) = 2y 2 = h0 (y) = h0 (y) = 2y 2 = h(y) = y 2 2y + C.
y
Hence,
t2 + 3t + y 2 2y = C

53
Problem 6.15
Consider the differential equation (ye2ty + t) + bte2ty y 0 = 0. Determine for
which value of b this equation is exact, and then solve it with this value of b.
Solution.
We have M (t, y) = ye2ty + t and N (t, y) = bte2ty . For the equation to be
exact we must have M
(t, y) = N
(t, y), that is,
y
t
e2ty + 2tye2ty = be2ty + 2ybte2ty .
Dividing through by e2ty to obtain
1 + 2ty = b + 2byt = b(1 + 2ty).
This implies b = 1. Hence, the equation is
(ye2ty + t) + te2ty y 0 = 0.
Now,
H
(t, y) = ye2ty + t = H(t, y) =
t

1
t2
(ye2ty + t)dt = e2ty + + h(y).
2
2

Also
H
(t, y) = te2ty = te2ty + h0 (y) = h0 (y) = 0 = h(y) = C.
y
Hence,
1 2ty t2
e + =C
2
2
Problem 6.16
Consider the differential equation y + (2t yey )y 0 = 0. Check that this
equation is not exact. Now multiply the equation by y. Check that the new
equation is exact, and solve it.
Solution.
If we let M (t, y) = y and N (t, y) = 2t yey we see that M
(t, y) = 1 and
y
N
(t, y) = 2 so that the equation is not exact. If we multiply the given
t
equation by y then M (t, y) = y 2 and N (t, y) = 2ty y 2 ey . In this case,

54

6 EXACT DIFFERENTIAL EQUATIONS

M
(t, y)
y

N
(t, y)
t

= 2y so that the equation is exact.

Now,
H
(t, y) = y 2 = H(t, y) =
t
Also

y 2 dt = ty 2 + h(y).

H
(t, y) = 2ty y 2 ey = 2ty + h0 (y) = h0 (y) = y 2 ey .
y

Using integration by parts twice we find


h(y) = y 2 ey + 2yey 2ey + C.
Hence,
ty 2 y 2 ey + 2yey 2ey = C
Problem 6.17
(a) Consider the differential equation
y 0 + p(t)y = g(t)
with p(t) 6= 0. Show
that this equation is not exact.
R
p(t)dt
(b) Let (t) = e
. Show that the equation
(t)(y 0 + p(t)y) = (t)g(t)
is exact and solve it.
Solution.
(t, y) = p(t) 6= 0
(a) We have M (t, y) = p(t)y g(t) and N (t, y) = 1. Since M
y
N
and t (t, y) = 0, the differential equation is not exact.
(b) Here, we have M (t, y) =R (t)p(t)y (t)g(t) and N (t, y) = (t). Thus,
M
(t, y) = N
(t, y) = p(t)e p(t)dt . That is, the new differential equation is
y
t
exact.
Now,
H
(t, y) =(t)p(t)y (t)g(t)
t
Z
H(t, y) =

((t)p(t)y (t)g(t))dt = (t)y

Z
(t)g(t)dt + h(y).

55
Also
H
(t, y) = (t) = (t) + h0 (y) = h0 (y) = 0 = h(y) = C.
y
Hence,
Z
(t)y
and so

y(t) = e

p(t)dt

Z
e

(t)g(t) = C
R

p(t)dt

g(t)dt + Ce

p(t)dt

Problem 6.18
Use the method of the previous problem to solve the linear, first-order equation y 0 yt = 1, with initial condition y(1) = 7. First, check that this equation
is not exact. Next, find (t). Multiply the equation by (t) and check that
the new equation is exact. Solve it, using the method of exact equations.
Solution.
For the given equation we have M (t, y) = 1 + yt and N (t, y) = 1. Since
R
M
1
N
dt
t =
(t,
y)
=
and
(t,
y)
=
0,
the
equation
is
not
exact.
Let
(t)
=
e
y
t
t
1
. Multiply the given equation by (t) to obtain
t
y 1
1
(1 + )( ) y 0 = 0.
t t
t
In this equation, M (t, y) = (1 + yt )( 1t ) and N (t, y) = 1t . Also, M
(t, y) =
y
N
1
(t, y) = t2 so that the new equation is exact. By the previous exercise the
t
solution is given by
Z
1
y(t) = t
dt + Ct = t ln t + Ct.
t
Since y(1) = 7 we find C = 7. Hence, y(t) = t ln t + 7t
Problem 6.19
Put the following differential equation in the Exact Differential Equation
form and find the general solution
y0 =

y 3 2ty
.
t2 3ty 2

56

6 EXACT DIFFERENTIAL EQUATIONS

Solution.
Rewriting this equation in the form
(y 3 2ty) + (3ty 2 t2 )y 0 = 0
we find M (t, y) = y 3 2ty and N (t, y) = 3ty 2 t2 . Also, notice that M
(t, y) =
y
N
(t, y) = 3y 2 2t. Now,
t
Z
H
3
(t, y) = y 2ty = H(t, y) = (y 3 2ty)dt = ty 3 t2 y + h(y).
t
Also
H
(t, y) = 3ty 2 t2 = 3ty 2 t2 + h0 (y) = h0 (y) = 0 = h(y) = C.
y
Hence,
ty 3 t2 y = C
Problem 6.20
The following differential equations are exact. Solve them by that method.
(a) (4t3 y + 4t + 4)y 0 = 8 4y 6t2 y 2 , y(1) = 1.
(b) (6 4y + 16t) + (10y 4t + 2)y 0 = 0, y(1) = 2.
Solution.
(a) We have M (t, y) = 6t2 y 2 + 4y 8 and N (t, y) = 4t3 y + 4t + 4. Notice
(t, y) = N
(t, y) = 12t2 y + 4. Now,
that M
y
t
Z
H
2 2
(t, y) = 6t y +4y8 = H(t, y) = (6t2 y 2 +4y8)dt = 2t3 y 2 +4ty8t+h(y).
t
Also
H
(t, y) = 4t3 y + 4t + 4 = 4t3 y + 4t + h0 (y) = h0 (y) = 4 = h(y) = 4y + C.
y
Hence,
2t3 y 2 + 4ty 8t + 4y = C.
Since y(1) = 1 we find C = 6. Hence, 2t3 y 2 + 4ty 8t + 4y = 6
(b) We have M (t, y) = 6 4y + 16t and N (t, y) = 10y 4t + 2. Notice that
M
(t, y) = N
(t, y) = 4. Now,
y
t
Z
H
(t, y) = 64y+16t = H(t, y) = (64y+16t)dt = 6t4ty+8t2 +h(y).
t

57
Also
H
(t, y) = 10y4t+2 = 4t+h0 (y) = h0 (y) = 10y+2 = h(y) = 5y 2 +2y+C.
y
Hence,
6t 4ty + 8t2 + 5y 2 + 2y = C.
Since y(1) = 2 we find C = 30. Hence, 6t 4ty + 8t2 + 5y 2 + 2y = 30

58

6 EXACT DIFFERENTIAL EQUATIONS

7 Substitution Techniques:
Bernoulli and Riccati Equations
Problem 7.1
Solve the Bernoulli equation
y0 =

t2 + 3y 2
, t > 0.
2ty

Solution.
The given equation can be written in the form
y0

3
1
y = ty 1 .
2t
2

Divide through by y 1 to obtain


yy 0

t
3 2
y = .
2t
2

Let z = y 2 . Then the last equation becomes


3
z0 z = t
t
and this is a linear first order differential equation.
To solve this equation, we use the integrating factor method. Let (t) = t3 .
Then
Z
3
z(t) = t
t3 tdt + Ct3 = t2 + Ct3 .
The general solution to the initial problem is implicitly defined by
y 2 = t2 + Ct3
59

607 SUBSTITUTION TECHNIQUES: BERNOULLI AND RICCATI EQUATIONS


Problem 7.2
2
Find the general solution of y 0 + ty = tet y 3 .
Solution.
Divide the given equation by y 3 to obtain
2

y 3 y 0 + ty 4 = tet .
Let z = y 4 so that the previous equation becomes
2

z 0 + 4tz = 4tet .
2

The integrating factor is (t) = e2t . Thus,


Z
2
2
2
2
2
2t2
z(t) = e
e2t 4tet dt + Ce2t = 2et + Ce2t .
Finally, the general solution to the original equation is defined implicitly by
the equation
2
2
y 4 = 2et + Ce2t
Problem 7.3
Solve the IVP ty 0 + y = t2 y 2 , y(0.5) = 0.5.
Solution.
Divide through by y 2 to obtain
ty 2 y 0 + y 1 = t2
or

1
y 2 y 0 + y 1 = t.
t

Let z = y 1 so that
1
z z = t, z
t
0

 
1
= 2.
2

Solving this equation by the integrating factor method with (t) =


Z
1
z(t) = t
(t)dt + Ct = t2 + Ct = t(C t).
t
Hence, y(t) =

1
.
t(Ct)

But y( 12 ) =

1
2

y(t) =

so that C = 4.5. Thus,


1
t(4.5 t)

1
t

we find

61
Problem 7.4
Solve the IVP y 0 1t y = y 2 , y(1) = 1, t > 0.
Solution.
Divide through by y 2 to obtain
1
y 2 y 0 y 1 = 1.
t
So let z = y 1 . Thus,
1
z 0 + z = 1, z(1) = 1.
t
Solving this equation using the integrating factor method with (t) = t we
find
Z
1
t
z(t) =
tdt + Ct1 = + Ct1 .
t
2
Since z(1) = 1, we find C = 21 . Hence, z = 12 (t + 1t ) and y(t) =

2t
t2 +1

Problem 7.5
Solve the IVP y 0 = y(1 y), y(0) = 21 .
Solution.
Rewriting the given equation in the form y 0 y = y 2 . Divide through by
y 2 to obtain
y 2 y 0 y 1 = 1.
Let z = y 1 . Then
z 0 + z = 1, z(0) = 2.
Solving this equation using the integrating factor method with (t) = et we
obtain
Z
t
et dt + Cet = 1 + Cet .
z(t) = e
But z(0) = 2 so that C = 1 and thus z(t) = 1 + et . Finally, y(t) =
(1 + et )1
Problem 7.6
Solve y 0 + y = ty 4 .

627 SUBSTITUTION TECHNIQUES: BERNOULLI AND RICCATI EQUATIONS


Solution.
Divide through by y 4 to obtain
y 4 y 0 + y 3 = t.
Let z = y 3 so that
z 0 3z = 3t.
Solving this equation using the integrating factor method with (t) = e3t
we find
Z
1
3t
z(t) = e
e3t (3t)dt + Ce3t = t + + Ce3t .
3
1

Hence, y(t) = (t + 13 + Ce3t ) 3


Problem 7.7
Solve the differential equation y 0 = 1 + t2 y 2 given that y1 (t) = t is a
particular solution.
Solution.
0
Let z1 = y t. Then zz2 = y 0 1. Substituting we find

2
1
z0
2
+t .
2 +1=1+t
z
z
Simplifying this last equation to obtain
z 0 2tz = 1.
2

Solving this equation by the method of integrating factor with (t) = et


we find
Z t
2
2
t2
es ds + Cet .
z(t) = e
0

The general solution to the differential equation is


Z t
2
2
t2
y(t) = (e
es ds + Cet )1 + t
0

Problem 7.8
Perform a change of variable that changes the Bernoulli equation y 0 +y +y 2 =
0 into a linear equation in the new variable. Do NOT try to solve the equation
or proceed further than with any calculations.

63
Solution.
Dividing through by y 2 to obtain
y 2 y 0 + y 1 = 1.
Letting z = y 1 to obtain
z0 z = 1
Problem 7.9
Consider the equation
y 0 = y y 3 ,  > 0, > 0.
(a) Use the Bernoulli transformation to change this nonlinear equation into
a linear equation.
(b) Solve the resulting linear equation in part (a) and use the solution to find
the solution of the given differential equation above.
Solution.
(a) Dividing through by y 3 to obtain
y 3 y 0 y 2 = .
Letting z = y 2 to obtain
z 0 + 2z = 2.
(b) Using the method of integrating factor with (t) = e2t we find
Z

2t
z(t) = e
e2t 2dt + Ce2t = + Ce2t .

Hence,
y(t) = (

+ Ce2t ) 2


Problem 7.10
Consider the differential equation
y0 = f

y 

.
t
leads to a separable differential equation

(a) Show that the substitution z = yt


in z.
(b) Use the above method to solve the initial-value problem
y0 =

t+y
, y(1) = 0.
ty

647 SUBSTITUTION TECHNIQUES: BERNOULLI AND RICCATI EQUATIONS


Solution.
(a) Letting z =

y
t

then y = tz. Thus, y 0 = z + tz 0 . Hence,


tz 0 + z = f (z)

or

z0
1
=
f (z) z
t
which is a separable differential equation.
(b) Note first that
1 + yt
t+y
=
.
ty
1 yt

Letting z =

y
t

we obtain

1+z
1z
1 + z2
tz 0 =
1z
1z 0 1
z =
1 + z2
t
Z
Z
Z
zz 0
dt
z0
dt

dt
=
1 + z2
1 + z2
t
1
arctan z ln (1 + z 2 ) = ln |t| + C
2
2 arctan z = ln t2 (1 + z 2 ) + C

y 
 y 2 
2
= ln t 1 +
2 arctan
+C
t
t
tz 0 + z =

Since y(1) = 0, C = 0. Hence,



y 
 y 2 
2
2 arctan
= ln t 1 +
t
t
Problem 7.11
Solve: y 0 + y3 = et y 4 .
Solution.
Divide through by y 4 to obtain y 4 y 0 + 31 y 3 = et . Letting z = y 3 to obtain
z 0 z = 3et .

65
Solving this equation by the method of integrating factor with (t) = et we
find
Z
z(t) = et

et (3et )dt + Cet = 3tet + Cet .

Hence,
1

y(t) = (3tet + Cet ) 3


Problem 7.12
Solve: ty 0 + y = ty 3 .

Solution.
Dividing through by ty 3 to obtain y 3 y 0 + 1t y 2 = 1. Letting z = y 2 to
obtain
2
z 0 z = 2.
t
Solving this equation by the method of integrating factor with (t) =
find
Z
1
2
z(t) = t
(2)dt + Ct2 = 2t + Ct2 .
2
t

1
t2

we

Hence,
1

y(t) = (2t + Ct2 ) 2


Problem 7.13
Solve: ty 0 + y = t2 y 2 ln t.

Solution.
Dividing through by ty 2 to obtain y 2 y 0 + 1t y 1 = t ln t. Letting z = y 1 to
obtain
1
z 0 z = t ln t.
t
Solving this equation by the method of integrating factor with (t) =
find
Z
z(t) = t ( ln t)dt + Ct = t2 ln t + t2 + Ct.
Hence,
y(t) = (t2 ln t + t2 + Ct)1

1
t

we

667 SUBSTITUTION TECHNIQUES: BERNOULLI AND RICCATI EQUATIONS


Problem 7.14
Verify that y1 (t) = 2 is a particular solution to the Riccati equation
y 0 = 2 y + y 2 ,
and then find the general solution.
Solution.
Since y10 = 0 and 2 y1 + y12 = 2 2 + 4 = 0, we have y10 = 2 y1 + y12 .
Now, to solve the equation we let z1 = y 2. Substituting this into the above
equation to obtain
z 0 + 3z = 1.
Solving this equation by the method of integrating factor with (t) = e3t we
find
Z
1
3t
z(t) = e
e3t dt + Ce3t = + Ce3t .
3
Hence,
1
y(t) = ( + Ce3t )1 + 2
3
Problem 7.15
Verify that y1 (t) =

2
t

is a particular solution to the Riccati equation


y0 =

1
4
y + y2,
2
t
t

and then find the general solution.


Solution.
Since y10 = t22 and t42 1t y1 + y12 = t22 , y1 is a solution to the differential
equation. Next, let z 1 = y 2t then substituting into the previous equation
we find
3
z 0 + z = 1.
t
Solving this equation by the method of integrating factor with (t) = t3 we
find
Z
t
3
z(t) = t
t3 dt + Ct3 = + Ct3 .
4
Hence,
t
2
y(t) = ( + Ct3 )1 +
4
t

8 Graphical Solution: Direction


Field of y 0 = f (t, y)
Problem 8.1
Sketch the direction field for the differential equation in the window 3
t 3, 3 y 3.
(a) y 0 = y (b) y 0 = t y.

Solution.
(a)

(b)
67

68

8 GRAPHICAL SOLUTION: DIRECTION FIELD OF Y 0 = F (T, Y )

Problem 8.2
Match each direction field with the equation that the slope field could represent. Each direction field is drawn in the portion of the ty-plane defined by
6 t 6, 4 y 4.
(a) y 0 = t (b) y 0 = sin t (c) y 0 = 1 y (d) y 0 = y(2 y).

Solution.
(A) y 0 = sin t

(B) y 0 = y(2 y)

(c) y 0 = t

Problem 8.3
State whether or not the equation is autonomous.

(D) y 0 = 1 y

69
(a) y 0 = t

(b) y 0 = sin t

Solution.
(a) No (b) No

(c) Yes

(c) y 0 = 1 y

(d) y 0 = y(2 y).

(d) Yes

Problem 8.4
Find all the equilibrium solutions of each of the autonomous differential equations below
(a) y 0 = (y 1)(y 2).
(b) y 0 = (y 1)(y 2)2 .
(c) y 0 = (y 1)(y 2)(y 3).

Solution.
(a) y(t) 1, y(t) 2.
(b) y(t) 1, y 2.
(c) y 1, y 2, y 3
Problem 8.5
Find an autonomous differential equation with an equilibrium solution at
y = 1 and satisfying y 0 < 0 for < y < 1 and 1 < y < .
Solution.
One answer is the differential equation: y 0 = (y 1)2
Problem 8.6
Find an autonomous differential equation with no equilibrium solutions and
satisfying y 0 > 0.
Solution.
Consider the differential equation y 0 = C where C is a positive constant
Problem 8.7
Find an autonomous differential equation with equilibrium solutions y = n2 ,
where n is an integer.
Solution.
One answer is the DE y 0 = sin (2y)

70

8 GRAPHICAL SOLUTION: DIRECTION FIELD OF Y 0 = F (T, Y )

Problem 8.8
Find an autonomous differential equation with equilibrium solutions y = 0
and y = 2 and satisfying the properties y 0 > 0 for 0 < y < 2; y 0 < 0 for y < 0
or y > 2.

Solution.
An answer is y 0 = y(2 y)

Problem 8.9
Classify whether the equilibrium solutions are stable, unstable, or neither.
(a) y 0 = 1 y 2 .
(b) y 0 = (y + 1)2 .

Solution.
Using the direction fields shown below we find
(a) y = 1 stable, y = 1 unstable

(b) y = 1 is neither. This is a semi-stable equilibrium

71

Problem 8.10
Consider the direction field below. Classify the equilibrium points, as asymptotically stable, semi-stable, or unstable.

72

8 GRAPHICAL SOLUTION: DIRECTION FIELD OF Y 0 = F (T, Y )

Solution.
The equilibrium solution at y = 1 is asymptotically stable where as the
equilibrium solution at y = 0 is unstable

Problem 8.11
Sketch the direction field of the equation y 0 = y 3 . Sketch the solution satisfying the condition y(1) = 1.

Solution.
As shown in the figure below, the domain of the solution is the interval
< t < 2

Problem 8.12
Find the equilibrium solutions and determine their stability
y 0 = y 2 (y 2 1), y(0) = y0 .

Solution.
The direction field is given below.

73

The equilibrium solution y = 1 is unstable; y = 0 is semi-stable; y = 1 is


asymptotically stable

Problem 8.13
Find the equilibrium solutions of the equation

y 0 = y 2 4y

then decide whether they are asymptotically stable, semi-stable, or unstable.


What is the long-run behavior if y(0) = 5?y(0) = 4?y(0) = 3?

Solution.
The direction field is given below.

74

8 GRAPHICAL SOLUTION: DIRECTION FIELD OF Y 0 = F (T, Y )

The equilibrium solution y = 4 is unstable while y = 0 is asymptotically


stable. If y(0) = 5 then limt y(t) = . If y(0) = 4 then limt y(t) = 4.
If y(0) = 3 then limt y(t) = 0

Problem 8.14
What is limt y(t) for the initial-value problem

y 0 = sin (y(t)), y(0) =

.
2

Solution.
y(t) = 0 and y(t) = are two equilibrium solutions. According to the
direction field shown below we conclude that

lim y(t) =

75

76

8 GRAPHICAL SOLUTION: DIRECTION FIELD OF Y 0 = F (T, Y )

9 Numerical Solutions to
ODEs: Eulers Method and its
Variants
In Problems 9.1 - 9.3, answer the following questions:
(a) Solve the initial value problem analytically, using an appropriate solution
technique.
(b) For the given initial value problem express yn+1 in terms of yn using
Heuns method.
(c) For the given initial value problem express yn+1 in terms of yn using the
Modified Eulers method.
(d) Use a step size h = 0.1. Compute the first three approximations y1 , y2 , y3
using the method in part (b).
(e) Use a step size h = 0.1. Compute the first three approximations y1 , y2 , y3
using the method in part (c).
(f) For comparison, calculate and list the exact solution values y(t1 ), y(t2 ), y(t3 ).
Problem 9.1
y 0 = 2t 1, y(1) = 0.
Solution.
(a) Integrating y 0 = 2t 1 we find y(t) = t2 t + C. Imposing the initial
condition y(1) = 0, we obtain y(t) = t2 t.
(b) Since f (t, y) = 2t 1, it follows that f (t + h, y + hf (t, y)) = 2(t + h) 1.
Therefore, Heuns method takes the form
h
yn+1 = yn + [(2tn 1) + (2tn+1 1)].
2
77

789 NUMERICAL SOLUTIONS TO ODES: EULERS METHOD AND ITS VARIANTS


(c) As in part (b), we find the modified Eulers method takes the form
h
yn+1 = yn + h[2(tn + ) 1].
2
(d)
n
0
1
2
3

tn
yn
1.0000 0
1.1000 0.1100
1.2000 0.2400
1.3000 0.3900

n
0
1
2
3

tn
yn
1.0000 0
1.1000 0.1100
1.2000 0.2400
1.3000 0.3900

n
0
1
2
3

tn
yn
1.0000 0
1.1000 0.1100
1.2000 0.2400
1.3000 0.3900

(e)

(f)

Problem 9.2
y 0 = y, y(0) = 1.
Solution.
(a) Integrating y 0 = y and imposing the initial condition we find y(t) = et .
(b) Heuns method takes the form yn+1 = (1 h + 0.5h2 )yn .
(c) The modified Eulers method takes the form yn+1 = (1 h + 0.5h2 )yn .
(d)
n
0
1
2
3

tn
yn
0.0000 1.0000
0.1000 0.9050
0.2000 0.8190
0.3000 0.7412

79
(e)
n
0
1
2
3

tn
yn
0.0000 1.0000
0.1000 0.9050
0.2000 0.8190
0.3000 0.7412

n
0
1
2
3

tn
yn
0.0000 1.0000
0.1000 0.9048
0.2000 0.8187
0.3000 0.7408

(f)

Problem 9.3
y 2 y 0 + t = 0, y(0) = 1.
Solution.
(a) Solving the separable equation y 2 y 0 + t = 0 we find y 3 = 1.5t2 + C.
1
Imposing the initial condition y(0) = 1, we obtain y(t) = (1 1.5t2 ) 3 .
(b) Since f (t, y) = ty 2 , it follows that f (t + h, hf (t, y)) = (t + h)[y +
h(ty)]2 . Therefore, Heuns method takes the form
h
yn+1 = yn + [tn yn2 tn+1 (yn htn yn2 )2 ].
2
(c) The modified Eulers method takes the form
yn+1 = yn h(tn + 0.5h)(yn 0.5htn yn2 )2 .
(d)
n
0
1
2
3
(e)

tn
yn
0.0000 1.0000
0.1000 0.9950
0.2000 0.9796
0.3000 0.9529

809 NUMERICAL SOLUTIONS TO ODES: EULERS METHOD AND ITS VARIANTS


n
0
1
2
3

tn
yn
0.0000 1.0000
0.1000 0.9950
0.2000 0.9797
0.3000 0.9531

n
0
1
2
3

tn
yn
0.0000 1.0000
0.1000 0.9950
0.2000 0.9796
0.3000 0.9528

(f)

Problem 9.4
Consider the initial value problem
y 0 = 1 + y 2 , y(0) = 1.
(a) Find the exact solution of the given initial value problem.
(b) Use step size h = 0.05. Compute 20 steps of Eulers method, Heuns
method, and modified Eulers method. Compare the numerical values obtained at t = 1 by calculating the error |y(1) y20 |.
Solution.
(a) Solving the separable equation y0 = 1 + y 2 and imposing the initial
condition we obtain y(t) = tan t 4 .
(b)
n
0
1
2
3
4
..
.

tn
0.0000
0.0500
0.1000
0.1500
0.2000
..
.

Euler
1.0000
0.9000
0.8095
0.7267
0.6503
..
.

Heun
1.0000
0.9047
0.8177
0.7375
0.6630
..
.

Modified Euler
1.0000
0.9049
0.8179
0.7378
0.6634
..
.

20

1.0000

0.2355

0.2181

0.2173

The errors at t = 1 are, respectively, 0.0175, 1.419 104 , and 6.581 104

81
Problem 9.5
Consider the initial value problem
y 0 + 2y = 4, y(0) = 3.
(a) Find the exact solution of the given initial value problem.
(b) Use step size h = 0.05. Compute 20 steps of Eulers method, Heuns
method, and modified Eulers method. Compare the numerical values obtained at t = 1 by calculating the error |y(1) y20 |.
Solution.
(a) Solving the equation y 0 + 2y = 4 and imposing the initial condition we
obtain y(t) = 2 + e2t .
(b)
n
0
1
2
3
4
..
.

tn
0.0000
0.0500
0.1000
0.1500
0.2000
..
.

20

Euler Heun
3.0000 3.0000
2.9000 2.9050
2.8100 2.8190
2.7290 2.7412
2.6561 2.6708
..
..
.
.
1.0000 2.1216 2.1358

Modified Euler
3.0000
2.9050
2.8190
2.7412
2.6708
..
.
2.1358

The errors at t = 1 are, respectively, 1.3758 102 , 4.8717 104 , and


4.8717 104
In Problems 9.6 - 9.8, the given iteration is the result of applying N steps of
Eulers method, Heuns method, or the modified Eulers method to an initial
value problem of the form
y 0 = f (t, y), y(t0 ) = y0 , t0 t t0 + T.
Identify the numerical method and determine t0 , N, T, and f (t, y).
Problem 9.6
yn+1 = yn + h(yn + t2n yn3 ), y0 = 1
tn = 2 + nh, h = 0.02, n = 0, 1, 2, , 49.

829 NUMERICAL SOLUTIONS TO ODES: EULERS METHOD AND ITS VARIANTS


Solution.
Since tn = 2 + nh, h = 0.02, n = 0, 1, , 49, it follows that t0 = 2 and
N 1 = 49. Thus, N = 50 and T = tN t0 = 2 + N h 2 = 50(0.02) =
1. From the form of the iteration, it must be Eulers method. Therefore
f (t, y) = y + t2 y 3
Problem 9.7
yn+1





h
h
2
2
= yn + h tn +
sin yn + tn sin yn , y0 = 1
2
2
tn = nh, h = 0.01, n = 0, 1, 2, , 199.

Solution.
Since tn = nh, h = 0.01, n = 0, 1, , 199, it follows that t0 = 0 and
N 1 = 199. Thus, N = 200 and T = tN t0 = N h = 200(0.01) = 2. From
the form of the iteration, it must be the modified Eulers method. Therefore,
f (t, y) = t sin2 y
Problem 9.8
h
yn+1 = yn + [tn yn2 + 2 + (tn + h)(yn + h(tn yn2 + 1))2 ], y0 = 2
2
tn = 1 + nh, h = 0.05, n = 0, 1, 2, , 99.
Solution.
Since tn = 1 + nh, h = 0.05, n = 0, 1, , 99, it follows that t0 = 1 and
N 1 = 99. Thus, N = 100 and T = tN t0 = 1 + N h 1 = 100(0.05) =
5. From the form of the iteration, it must be Heuns method. Therefore
f (t, y) = ty 2 + 1

10 Second Order Linear


Differential Equations:
Existence and Uniqueness
Results
In Problems 10.1 - 10.6, determine the largest tinterval on which Theorem
15.1 guarantees the existence of a unique solution.
Problem 10.1
y 00 + y 0 + 3ty = tan t, y() = 1, y 0 () = 1.
Solution.
In this equation p(t) = 1, q(t) = 3t and g(t) = tan t. All three functions are
continuous for all t 6= (2n + 1) 2 , where n is an integer. With t0 = then the
largest interval of existence guaranteed by Theorem 15.1 is 2 < t < 3
2
Problem 10.2
et y 00 +

1
y
t2 1

= 4t , y(2) = 1, y 0 (2) = 2.

Solution.
In this equation p(t) = 0, q(t) = et (t211) , and g(t) = 4t et . All three functions
are continuous for all t 6= 1, 0, 1. With t0 = 2 then the largest interval of
existence guaranteed by Theorem 15.1 is < t < 1
83

8410 SECOND ORDER LINEAR DIFFERENTIAL EQUATIONS: EXISTENCE AND UNIQU


Problem 10.3
ty 00 +

sin 2t 0
y
t2 9

+ 2y = 0, y(1) = 0, y 0 (1) = 1.

Solution.
2t
In this equation p(t) = t(tsin2 9)
, q(t) = 2t , and g(t) = 0. All three functions
are continuous for all t 6= 3, 0, 3. With t0 = 1 then the largest interval of
existence guaranteed by Theorem 15.1 is 0 < t < 3
Problem 10.4
ty 00 (1 + t)y 0 + y = t2 e2t , y(1) = 0, y 0 (1) = 1.
Solution.
In this equation p(t) = 1+t
, q(t) = 1t , and g(t) = te2t . All three functions
t
are continuous for all t 6= 0. With t0 = 1 then the largest interval of
existence guaranteed by Theorem 15.1 is < t < 0
Problem 10.5
(sin2 t)y 00 (2 sin t cos t)y 0 + (cos2 t + 1)y = sin3 t, y( 4 ) = 0, y 0 ( 4 ) =

2.

Solution.
2
t
, and g(t) = sin t. All three
In this equation p(t) = 2 cos
, q(t) = cossin2t+1
sin t
t
functions are continuous for all t 6= n, where n is an integer. With t0 = 4
then the largest interval of existence guaranteed by Theorem 15.1 is 0 < t <

Problem 10.6
t2 y 00 + ty 0 + y = sec (ln t), y( 3 ) = 0, y 0 ( 3 ) = 1.
Solution.
t)
In this equation p(t) = 1t , q(t) = t12 , and g(t) = sec t(ln
. All three functions
2

are continuous for all t > 0 and t 6= e(2n+1) 2 , where n is an integer. Note that
sec ln t = 0 only if cos ln t = 0 which implies that ln t = (2n + 1)) 2 . With

85
t0 = 3 then the largest interval of existence guaranteed by Theorem 15.1 is

e 2 < t < e 2
In Problems 10.7 - 10.8, give an example of an initial value problem of the
form y 00 + p(t)y 0 + q(t)y = 0, y(t0 ) = y0 , y 0 (t0 ) = y00 for which the given
tinterval is the largest on which Theorem 15.1 guarantees a unique solution.
Problem 10.7

< t < .
Solution.
One such an answer is
y 00 + y 0 + y = 0, y(0) = 0, y 0 (0) = 1
Problem 10.8

3 < t < .
Solution.
One such an answer is
y 00 +

1
y0
t3

+ y = 1, y(4) = 0, y 0 (4) = 1

Problem 10.9
Determine the largest interval in which the initial-value problem
(t 3)y 00 + ty 0 + (ln |t|)y = 0, y(1) = 0, y 0 (1) = 1
is certain to have a unique solution.
Solution.
|t|
t
We have p(t) = t3
and q(t) = lnt3
. Both functions are continuous for all
t 6= 0, 3. Since t0 = 1, the largest t-interval is 0 < t < 3

8610 SECOND ORDER LINEAR DIFFERENTIAL EQUATIONS: EXISTENCE AND UNIQU


Problem 10.10
Theorem 15.1 tells us that the initial-value problem
y 00 + t2 y = 0, y(0) = y 0 (0) = 0
defines exactly one function y(t). Using only Theorem 15.1, show that this
function has the additional property y(t) = y(t). That is, y(t) is an even
function.
Solution.
Let Y (t) = y(t). Then Y 00 + t2 Y = y 00 + t2 y = 0, Y (0) = Y 0 (0) = 0 so that
Y (t) is a solution to the given initial-value problem. By Theorem 15.1, we
must have Y (t) = y(t), i.e., y(t) = y(t) for all real numbers t

11 The General Solution of 2nd


Order Linear Homogeneous
Equations
In Problems 11.1-11.7, the t-interval of solution is < t < unless indicated otherwise.
(a) Determine whether the given functions are solutions to the differential
equation.
(b) If both functions are solutions, calculate the Wronskian. Does this calculation show that the two functions form a fundamental set of solutions?
(c) If the two functions have been shown in (b) to form a fundamental set,
construct the general solution and determine the unique solution satisfying
the initial value problem.
Problem 11.1
y 00 4y = 0, y1 (t) = e2t , y2 (t) = 2e2t , y(0) = 1, y 0 (0) = 2.
Solution.
(a)
y100 4y1 = 4e2t 4e2t = 0
y200 4y2 = 8e2t 8e2t = 0
so both functions are solutions.
(b)
2t
e
2e2t
W (y1 (t), y2 (t)) = 2t
2e 4e2t
87



= 8 6= 0

8811 THE GENERAL SOLUTION OF 2ND ORDER LINEAR HOMOGENEOUS EQUATIONS


so {y1 , y2 } is a fundamental set of solutions.
(c) We have y(t) = c1 e2t + 2c2 e2t and y 0 (t) = 2c1 e2t 4c2 e2t . The initial
conditions imply c1 + 2c2 = 1 and 2c1 4c2 = 2. Solving we find c1 = 0
and c2 = 21 . Hence, y(t) = e2t
Problem 11.2
y 00 + y = 0, y1 (t) = sin t cos t, y2 (t) = sin t, y( 2 ) = 1, y 0 ( 2 ) = 1.
Solution.
(a)
y100 + y1 = 2 cos t sin t + 2 cos t sin t + sin t cos t 6= 0
so y1 is not a solution.
y200 + y2 = sin t + sin t = 0
so y2 is a solution
Problem 11.3
y 00 4y 0 + 4y = 0, y1 (t) = e2t , y2 (t) = te2t , y(0) = 2, y 0 (0) = 0.
Solution.
(a)
y100 4y10 + 4y1 = 4e2t 8e2t + 4e2t = 0.
y200 4y20 + 4y2 = 4e2t + 4te2t 4e2t 8te2t + 4te2t = 0
so both functions are solutions.
(b)
2t
e
te2t
W (y1 (t), y2 (t)) = 2t
2e (2t + 1)e2t



= e4t 6= 0

so {y1 , y2 } is a fundamental set of solutions.


(c) We have y(t) = c1 e2t + c2 te2t and y 0 (t) = 2c1 e2t + (c2 + 2c2 t)e2t . The initial
conditions imply c1 = 2 and c2 = 4. Hence, y(t) = 2e2t 4te2t

89
Problem 11.4
ty 00 + y 0 = 0, y1 (t) = ln t, y2 (t) = ln 3t, y(3) = 0, y 0 (3) = 3, 0 < t < .
Solution.
(a)
ty100 + y10 = tt2 +

1
t

=0

ty200 + y20 = tt2 +

1
t

=0

so both functions are solutions.


(b)

ln t ln (3t)
W (y1 (t), y2 (t)) = 1
1
t



= 1 ln 3 6= 0

t

so {y1 , y2 } is a fundamental set of solutions.


(c) We have y(t) = c1 ln t+c2 ln (3t) and y 0 (t) = ct1 + ct2 . The initial conditions
imply c1 + 2c2 = 0 and c1 + c2 = 9. Solving we find c1 = 18 and c2 = 9.
Hence, y(t) = 18 ln t 9 ln (3t), t > 0
Problem 11.5
t2 y 00 ty 0 3y = 0, y1 (t) = t3 , y2 (t) = t1 , y(1) = 0, y 0 (1) = 2, t < 0.
Solution.
(a)
t2 y100 ty10 3y1 = t2 (6t) t(3t2 ) 3t3 = 0
t2 y200 ty20 3y2 = t2 (2t3 ) t(t2 ) 3(t1 ) = 0
so both functions are solutions.
(b)
3
t t1
W (y1 (t), y2 (t)) = 2
3t
t2



= 4t 6= 0, t < 0

so {y1 , y2 } is a fundamental set of solutions.


(c) We have y(t) = c1 t3 c2 t1 and y 0 (t) = 3c1 t2 +c2 t2 . The initial conditions
imply c1 + c2 = 0 and 3c1 + c2 = 2. Solving we find c1 = 21 and c2 = 21 .
Hence, y(t) = 12 (t1 t3 ), t > 0

9011 THE GENERAL SOLUTION OF 2ND ORDER LINEAR HOMOGENEOUS EQUATIONS


Problem 11.6
y 00 = 0, y1 (t) = t + 1, y2 (t) = t + 2, y(1) = 4, y 0 (1) = 1.
Solution.
(a) Since y100 = y200 = 0, both functions are solutions.
(b)


t + 1 t + 2
= 3 6= 0
W (y1 (t), y2 (t)) =
1
1
so {y1 , y2 } is a fundamental set of solutions.
(c) We have y(t) = c1 (t + 1) + c2 (t + 2) and y 0 (t) = c1 c2 . The initial
conditions imply 2c1 + c2 = 4 and c1 c2 = 1. Solving we find c1 = 1 and
c2 = 2. Hence, y(t) = t + 5
Problem 11.7
t

4y 00 + 4y 0 + y = 0, y1 (t) = e 2 , y2 (t) = te 2 , y(1) = 1, y 0 (1) = 0.


Solution.
(a)
t

4y100 + 4y10 + y1 = 4e 2 6= 0
so y1 is not a solution.
t

4y200 + 4y20 + y2 = 8e 2 + 4te 2 6= 0


so y2 is not a solution
Problem 11.8
The functions y1 (t) = t and y2 (t) = t ln t form a fundamental set of solutions
to the differential equation
t2 y 00 ty 0 + y = 0, 0 < t < .
(a) Show that y(t) = 2t + t ln 3t is a solution to the differential equation.
(b) Find c1 and c2 such that y(t) = c1 y1 (t) + c2 y2 (t).

91
Solution.
(a) t2 y 00 ty 0 + y = t2 t1 t(3 + ln (3t)) + 2t + t ln (3t) = 0.
(b) We have
c1 t + c2 t ln t
= 2t + t ln (3t)
c1 + c2 (1 + ln t) = 3 + ln (3t)
Using the elimination method we find c1 = 2 + ln 3 and c2 = 1. Thus,
y(t) = (2 + ln 3)t + t ln t
Problem 11.9
The functions y1 (t) = e3t and y2 (t) = e3t are known to be solutions of
y 00 + y 0 + y = 0, where and are constants. Determine and .
Solution.
Since y100 + y10 + y1 = 0, 3 + = 9. Since y200 + y20 + y2 = 0 we find
3 + = 9. Hence, = 0 and = 9
Problem 11.10
The functions y1 (t) = t and y2 (t) = et are known to be solutions of y 00 +
p(t)y 0 + q(t)y = 0.
(a) Determine the functions p(t) and q(t).
(b) On what tintervals are the functions p(t) and q(t) continuous?
(c) Compute the Wronskian of these two functions. On what tintervals is
the Wronskian nonzero?
(d) Are the observations in (b) and (c) consistent with Abels Theorem?
Solution.
(a) Since y100 +p(t)y10 +q(t)y1 = 0, p(t)+tq(t) = 0. Since y200 +p(t)y20 +q(t)y2 = 0
t
we find p(t) + q(t) = 1. Solving for p(t) and q(t) we find p(t) = t1
and
1
q(t) = t1 .
(b) Both p(t) and q(t) are continuous on (, 1) (1, ).
(c)


t et
= et (t 1).
W (y1 (t), y2 (t)) =
1 et
The Wronskian is nonzero for all t 6= 1.
(d) Yes. W 6= 0 on the two intervals on which p and q are both continuous
Problem 11.11
It is known that two solutions of y 00 +ty 0 +2y = 0 has a Wronskian W (y1 (t), y2 (t))
that satisfies W (y1 (1), y2 (1)) = 4. What is W (y1 (2), y2 (2))?

9211 THE GENERAL SOLUTION OF 2ND ORDER LINEAR HOMOGENEOUS EQUATIONS


Solution.
From Abels Theorem, we have
W (y1 (t), y2 (t)) = W (y1 (1), y2 (1))e

Rt
1

sds

t2

= 4e 2 + 2 .

Hence, W (y1 (2), y2 (2)) = 4e1.5


Problem 11.12
The pair of functions {y1 , y2 } is known to form a fundamental set of solutions
of y 00 +y 0 +y = 0, where and are constants. One solution is y1 (t) = e2t ,
and the Wronskian formed by these two solutions is W (y1 (t), y2 (t)) = et .
Determine the constants and .
Solution.
Since y100 + y10 + y1 = 0, we have 2 + = 4. Since W (y1 (t), y2 (t)) = et ,
we find W 0 (t) = et . But W 0 + pW = 0 so that et + pet = 0. Hence,
p(t) = 1 = . Thus, = 4 2 = 6
Problem 11.13
2
The Wronskian of a pair of solutions of y 00 + p(t)y 0 + 3y = 0 is W (t) = et .
What is the coefficient function p(t)?
Solution.
2
2
Since W 0 = pW, we have 2tet = p(t)et so that p(t) = 2t
Problem 11.14
Prove that if y1 and y2 have maxima or minima at the same point in an
interval I, then they cannot be a fundamental set of solutions on that interval.
Solution.
Suppose for example that both functions have a same maximum at t0 . Then
y10 (t0 ) = y20 (t0 ) = 0. But
W (y1 (t0 ), y2 (t0 )) = y1 (t0 )y20 (t0 ) y10 (t0 )y2 (t0 ) = 0.
Thus, {y1 , y2 } is not a fundamental set
Problem 11.15
Without solving the equation, find the Wronskian of two solutions of Bessels
equation
t2 y 00 + ty 0 + (t2 2 )y = 0.

93
Solution.
By Abels Theorem
W (y1 (t), y2 (t)) = W (y1 (t0 ), y2 (t0 ))e

Rt

ds
t0 s

= t0

W (y1 (t0 ), y2 (t0 ))


t

Problem 11.16
If W (y1 , y2 ) = t2 et and y1 (t) = t then find y2 (t).
Solution.
By the quotient rule


y2
y1

0
=

W
= et .
2
y1

Thus, one possible answer is


y2 (t) = tet
Problem 11.17
The functions t2 and 1/t are solutions to a 2nd order, linear homogeneous
ODE on t > 0. Verify whether or not the two solutions form a fundamental
solution set.
Solution.
Finding the Wronskian
2
t
1
t1
W (t , ) =
2t t2
t
2



= 3 6= 0

so that {y1 , y2 } is a fundamental set


Problem 11.18
Show that t3 and t4 cant both be solutions to a differential equation of the
form y 00 + p(t)y 0 + q(t)y = 0 where p and q are continuous functions defined
on the real numbers.
Solution.
Suppose that t3 and t4 are both solutions. Since W (t) = t6 , we find W (1) = 1
and so {y1 , y2 } is a fundamental set. By Abels Theorem, W (t) 6= 0 for all
< t < . But W (0) = 0, a contradiction. Hence, t3 and t4 cant be
both solutions for the differential equation for < t <

9411 THE GENERAL SOLUTION OF 2ND ORDER LINEAR HOMOGENEOUS EQUATIONS


Problem 11.19
Suppose that t2 + 1 is the Wronskian of two solutions to the differential
equation y 00 + p(t)y 0 + q(t)y = 0. Find p(t).
Solution.
Since W 0 = p(t)W, we have 2t = p(t)(t2 + 1). Thus, p(t) = t22t+1

12 Second Order Linear


Homogeneous Equations with
Constant Coefficients: Distinct
Characterisitc Roots
Problem 12.1
Solve the initial value problem
y 00 + y 0 2y = 0, y(0) = 3, y 0 (0) = 3.
Describe the behavior of the solution y(t) as t and t .
Solution.
The characteristic equation r2 + r 2 = 0 has roots r = 1 and r = 2 so
that the general solution is given by
y(t) = c1 et + c2 e2t .
The initial conditions and y 0 (t) = c1 et 2c2 e2t lead to the system c1 +c2 = 3
and c1 2c2 = 3. Solving this system, we find c1 = 1 and c2 = 2. Hence,
the unique solution to the initial value problem is
y(t) = et + 2e2t .
limt y(t) = and limt y(t) =
Problem 12.2
Solve the initial value problem
y 00 4y 0 + 3y = 0, y(0) = 1, y 0 (0) = 1.
Describe the behavior of the solution y(t) as t and t .
95

9612 SECOND ORDER LINEAR HOMOGENEOUS EQUATIONS WITH CONSTANT COEF


Solution.
The characteristic equation r2 4r + 3 = 0 has roots r = 1 and r = 3 so that
the general solution is given by
y(t) = c1 et + c2 e3t .
The initial conditions and y 0 (t) = c1 et +3c2 e3t lead to the system c1 +c2 = 1
and c1 + 3c2 = 1. Solving this system, we find c1 = 2 and c2 = 1. Hence,
the unique solution to the initial value problem is
y(t) = 2et + e3t .
limt y(t) = 0 and limt y(t) = limt e3t 1

2
e2t

Problem 12.3
Solve the initial value problem
y 00 y = 0, y(0) = 1, y 0 (0) = 1.
Describe the behavior of the solution y(t) as t and t .
Solution.
The characteristic equation r2 1 = 0 has roots r = 1 and r = 1 so that
the general solution is given by
y(t) = c1 et + c2 et .
The initial conditions and y 0 (t) = c1 et c2 et lead to the system c1 + c2 = 1
and c1 c2 = 1. Solving this system, we find c1 = 0 and c2 = 1. Hence, the
unique solution to the initial value problem is
y(t) = et .
limt y(t) = and limt y(t) = 0
Problem 12.4
Solve the initial value problem
y 00 + 5y 0 + 6y = 0, y(0) = 1, y 0 (0) = 1.
Describe the behavior of the solution y(t) as t and t .

97
Solution.
The characteristic equation r2 + 5r + 6 = 0 has roots r = 2 and r = 3 so
that the general solution is given by
y(t) = c1 e2t + c2 e3t .
The initial conditions and y 0 (t) = 2c1 e2t 3c2 e3t lead to the system
c1 + c2 = 1 and 2c1 + 3c2 = 1. Solving this system, we find c1 = 2 and
c2 = 1. Hence, the unique solution to the initial value problem is
y(t) = 2e2t e3t .
limt y(t) = limt e3t (2et 1) = and limt y(t) = 0
Problem 12.5
Solve the initial value problem
y 00 4y = 0, y(3) = 0, y 0 (3) = 0.
Describe the behavior of the solution y(t) as t and t .
Solution.
The characteristic equation r2 4 = 0 has roots r = 2 and r = 2 so that
the general solution is given by
y(t) = c1 e2t + c2 e2t .
The initial conditions and y 0 (t) = 2c1 e2t 2c2 e2t lead to the system c1 e6 +
c2 e6 = 0 and 2c1 e6 2c2 e6 = 0. Solving this system, we find c1 = 0 and
c2 = 0. Hence, the unique solution to the initial value problem is y(t) 0.
limt y(t) = 0 and limt y(t) = 0
Problem 12.6
Solve the initial value problem
2y 00 3y 0 = 0, y(2) = 3, y 0 (2) = 0.
Describe the behavior of the solution y(t) as t and t .

9812 SECOND ORDER LINEAR HOMOGENEOUS EQUATIONS WITH CONSTANT COEF


Solution.
The characteristic equation 2r2 3r = 0 has roots r = 0 and r = 1.5 so that
the general solution is given by
y(t) = c1 e1.5t + c2 .
The initial conditions and y 0 (t) = 1.5c1 e1.5t lead to the system c1 e3 + c2 = 3
and c1 = 0. Solving this system, we find c2 = 3. Hence, the unique solution
to the initial value probem y(t) 3.
limt y(t) = 3 and limt y(t) = 3
Problem 12.7
Solve the initial value problem
y 00 + 4y 0 + 2y = 0, y(0) = 0, y 0 (0) = 4.
Describe the behavior of the solution y(t) as t and t .
Solution.

2
The characteristic
equation
r
+
4r
+
2
=
0
has
roots
r
=
2

2 and

r = 2 + 2 so that the general solution is given by

2)t

+ c2 e(2+ 2)t .

The initial conditions and y 0 (t) = c1 (2 2)e(2 2)t +c2


(2+ 2)e(2+ 2)t
lead to the system c1 + c2 =0 and (2 2)c1 + (2 + 2)c2 = 4. Solving
this system, we find c1 = 2 and c2 = 2. Hence, the unique solution to
the initial value problem is

y(t) = 2e(2 2)t + 2e(2+ 2)t .


limt y(t) = limt e(2 2)t [ 2 + 2e2 2t ] = and
limt y(t) = 0
y(t) = c1 e(2

Problem 12.8
Solve the initial value problem
2y 00 y = 0, y(0) = 2, y 0 (0) =

2.

Describe the behavior of the solution y(t) as t and t .

99
Solution.

The characteristic equation 2r2 1 = 0 has roots r = 22 and r =


that the general solution is given by

y(t) = c1 e

2
t
2

2
2

so

+ c2 e

2
t
2

The initial conditions and y 0 (t) = 22 c1 e 2 t 22 c2 e 2 t lead to the system


c1 +c2 = 2 and c1 c2 = 2. Solving this system, we find c1 = 0 and c2 = 2.
Hence, the unique solution to the initial value problem is
y(t) = 2e

2
t
2

limt y(t) = and limt y(t) = 0


Problem 12.9
Obtain the general solution to the differential equation y 000 5y 00 + 6y 0 = 0.
Solution.
Let u = y 0 . Then u0 = y 00 and u00 = y 000 so that the given equation becomes
u00 5u0 + 6u = 0.
The characteristic equation r2 5r + 6 = 0 has roots r = 2 and r = 3 so that
the general solution is given by
u(t) = c1 e2t + c2 e3t .
But y 0 (t) = u(t) so that
Z
y(t) =

u(t)dt = C1 e2t + C2 e3t + C3

Problem 12.10
A particle of mass m moves along the x-axis and is acted upon by a drag
force proportional to its velocity. The drag constant is denoted by k. If x(t)
represents the particle position at time t, Newtons law of motion leads to
the differential equation mx00 (t) = kx0 (t).
(a) Obtain the general solution to this second order linear differential equation.
(b) Solve the initial value problem if x(0) = x0 and x0 (0) = v0 .
(c) What is limt x(t)?

10012 SECOND ORDER LINEAR HOMOGENEOUS EQUATIONS WITH CONSTANT COE


Solution.
k
.
(a) The characteristic equation is mr2 +kr = 0 with roots r = 0 and r = m
Thus, the general solution is
k

x(t) = c1 + c2 e m t .
k

k
(b) The initial conditions and x0 (t) = m
c2 e m t lead to c1 = x0 +
m
c2 = k v0 . Hence,

m  m kt
x(t) = x0 + v0 v0 e m .
k
k

(c) limt x(t) = x0 +

m
v
k 0

and

m
v
k 0

Problem 12.11
Find a homogeneous second-order linear ordinary differential equation whose
general solution is y(t) = c1 e2t + c2 et .
Solution.
The roots for the characteristic equation are r = 2 and r = 1 so that
(r 2)(r + 1) = 0 and hence r2 r 2 = 0. The homogeneous equation is
then y 00 y 0 2y = 0
Problem 12.12
Find the general solution of the differential equation y 00 3y 0 4y = 0.
Solution.
The characteristic equation r2 3r 4 = 0 has roots r = 1 and r = 4.
Thus,
y(t) = c1 et + c2 e4t
Problem 12.13
Find the general solution of the differential equation y 00 + 4y 0 5y = 0.
Solution.
The characteristic equation r2 + 4r 5 = 0 has roots r = 1 and r = 5.
Thus,
y(t) = c1 et + c2 e5t
Problem 12.14
Find the general solution of the differential equation 3y 00 + 2y 0 + y = 0.

101
Solution.
The characteristic equation 3r2 + 2r + 1 = 0 has roots r = 1 and r = 31 .
Thus,
t
y(t) = c1 et + c2 e 3
Problem 12.15
Solve the initial-value problem: y 00 + 3y 0 4y = 0, y(0) = 1, y 0 (0) = 1.
Solution.
The characteristic equation r2 + 3r 4 = 0 has roots r = 1 and r = 4.
Thus,
y(t) = c1 et + c2 e4t .
The initial conditions and y 0 (t) = c1 et 4c2 e4t lead to the system c1 +c2 = 1
and c1 4c2 = 1. Solving this system, we find c1 = 53 and c2 = 52 . Thus,
1
y(t) = (3et + 2e4t )
5
Problem 12.16
Solve the initial-value problem: 2y 00 + 5y 0 3y = 0, y(0) = 2, y 0 (0) = 1.
Solution.
The characteristic equation 2r2 + 5r 3 = 0 has roots r = 3 and r = 21 .
Thus,
t
y(t) = c1 e3t + c2 e 2 .
t

The initial conditions and y 0 (t) = 3c1 e3t + c22 e 2 lead to the system c1 +c2 =
2 and 3c1 + c22 = 1. Solving this system, we find c1 = 0 and c2 = 2. Thus,
t

y(t) = 2e 2

10212 SECOND ORDER LINEAR HOMOGENEOUS EQUATIONS WITH CONSTANT COE

13 Characteristic Equations
with Repeated Roots
In Problems 13.1 - 13.5 answer the following questions.
(a) Obtain the general solution of the differential equation.
(b) Impose the initial conditions to obtain the unique solution of the initial
value problem.
(c) Describe the behavior of the solution as t and t .
Problem 13.1
5
9y 00 6y 0 + y = 0, y(3) = 2, y 0 (3) = .
3
Solution.
(a) The characteristic equation 9r2 6r + 1 = 0 has the roots r1 = r2 = 13 .
The general solution is then
t

y(t) = c1 e 3 + c2 te 3 .
t

(b) The initial conditions and y 0 (t) = c31 e 3 + c2 e 3 + c32 te 3 lead to the system
c1 + 3c2 = 2e1 and c1 + 6c2 = 5e1 . Solving this system, we find c1 = e1
and c2 = e1 . Thus, the unique solution is
t

y(t) = e 3 1 (1 t).
(c)
limt y(t) = limt

1t
t

e1 3

= limt
t

1
t

1/3e1 3
t

=0
t

Now, for large t we have t1 1 so that e 3 1 (t1) e 3 1 . Since e 3 1


t
as t , we find e 3 1 (t 1) as t . Hence,
103

104

13 CHARACTERISTIC EQUATIONS WITH REPEATED ROOTS


t

limt y(t) = limt e 3 1 (t 1) =


Problem 13.2
3
25y 00 + 20y 0 + 4y = 0, y(5) = 4e2 , y 0 (5) = e2 .
5
Solution.
(a) The characteristic equation 25r2 +20r+4 = 0 has the roots r1 = r2 = 25 .
The general solution is then
2t

2t

y(t) = c1 e 5 + c2 te 5

2t

2t

2t

(b) The initial conditions and y 0 (t) = 2c51 e 5 + c2 e 5 2c52 te 5 lead to the
system c1 + 5c2 = 4 and 2c1 + 5c2 = 3. Solving this system, we find c1 = 1
and c2 = 1. Thus, the unique solution is
2t

y(t) = e 5 (t 1).
(c)
limt y(t) = limt e

2t
5

(1 t) =

and
limt y(t) = limt e

2t
5

(t 1) = limt

1
2 2t
e5
5

=0

Problem 13.3
y 00 4y 0 + 4y = 0, y(1) = 4, y 0 (1) = 0.
Solution.
(a) The characteristic equation r2 4r + 4 = 0 has the roots r1 = r2 = 2.
The general solution is then
y(t) = c1 e2t + c2 te2t .
(b) The initial conditions and y 0 (t) = 2c1 e2t +c2 e2t +2c2 te2t lead to the system
c1 + c2 = 4e2 and 2c1 + 3c2 = 0. Solving this system, we find c1 = 12e2
and c2 = 8e2 . Thus, the unique solution is
y(t) = 4e2t2 (2t 3).
(c)

105
limt y(t) = limt

2t3
e22t

= limt

2
2e22t

=0

and
limt y(t) =
Problem 13.4

y 00 + 2 2y 0 + 2y = 0, y(0) = 1, y 0 (0) = 0.
Solution.

(a)The characteristic equation r2 + 2 2r + 2 = 0 has the roots r1 = r2 =


2. The general solution is then

+ c2 te 2t .

0
2t
2t
2t
(b) The initial conditions
and
y
(t)
=

2c
e
+
c
e

2c
te
1
2
2

lead to c1 = 1 and c2 = 2.Thus, the unique solution is

y(t) = e 2t + 2te 2t = e 2t (1 + 2t).


y(t) = c1 e

2t

(c) We have
lim y(t) = = .

Also,

1 + 2t
2

= lim = 0
lim y(t) = lim
t
t
t
e 2t
2e 2t

Problem 13.5

3y 00 + 2 3y 0 + y = 0, y(0) = 2 3, y 0 (0) = 3.
Solution.

(a) The characteristic equation 3r2 + 2 3r + 1 = 0 has the roots r = r1 =


r2 = 13 . The general solution is then
y(t) = c1 ert + c2 tert .

(b) The initial conditions and y 0 (t) = rc1 ert + c2 ert + rc2 tert lead to c1 = 2 3
and c2 = 5. Thus, the unique solution is

t
y(t) = e 3 (5t + 2 3).
(c)

106

13 CHARACTERISTIC EQUATIONS WITH REPEATED ROOTS


limt y(t) = =

and
limt y(t) = limt

2 3+5t
t
e 3

= limt

(1/ 3)e

t
3

=0

Problem 13.6
Find the general solution of y 00 6y 0 + 9y = 0.
Solution.
The characteristic equation r2 6r + 9 = 0 has double roots r1 = r2 = 3 so
the general solution is
y(t) = c1 e3t + c2 te3t
Problem 13.7
Find the general solution of 4y 00 4y 0 + y = 0.
Solution.
The characteristic equation 4r2 4r + 1 = 0 has double roots r1 = r2 =
the general solution is
t
t
y(t) = c1 e 2 + c2 te 2
Problem 13.8
Solve the initial-value problem: y 00 + y 0 +

y
4

1
2

so

= 0, y(0) = 2, y 0 (0) = 0.

Solution.
The characteristic equation r2 + r + 41 = 0 has double roots r1 = r2 = 12 so
the general solution is
t
t
y(t) = c1 e 2 + c2 te 2 .
Since y(0) = 2, we find c1 = 2. Since y 0 (0) = 0, we find c1 2c2 = 0. Solving
for c2 , we find c2 = 1. Hence, the unique solution is
t

y(t) = 2e 2 + te 2

14 Characteristic Equations
with Complex Roots
Problem 14.1
For any z = + i we define the conjugate of z to be the complex number
z = i. show that = 12 (z + z) and = 2i1 (z z).
Solution.
Adding z and z, we find 2 = z + z. Hence, = 12 (z + z). Next, subtracting
z from z, we find 2i = z z. Therefore, = 2i1 (z z)
Problem 14.2
Write each of the complex numbers in the form + i, where and are
real numbers.

1. 2ei 3 .
3
2. (2 i)ei 2 .

3. ( 2ei 6 )4 .
Solution.
Recall Eulers identity: e+i = e (cos + i sin ). Thus,

1. 2ei 3 = 2 cos ( 3 ) + 2i sin ( 3 ) = 1 + i 3.


2. We have
3

(2 i)ei 2 =2ei 2 iei 2


  
 
  
 
3
3
3
3
=2 cos
+ i sin
i cos
+ i sin
2
2
2
2
=2(0 i) i(0 i)
= 1 2i.

2
3. ( 2ei 6 )4 = ( 2)4 ei 3 = 4( 12 + i 23 ) = 2 + 2i 3
107

108

14 CHARACTERISTIC EQUATIONS WITH COMPLEX ROOTS

Problem 14.3
Write each function in the form Aet cos t + iBet sin t, where , , A, and
B arereal numbers.
1. 2ei 2t .
2. 12 e2t+i(t+) .

3. ( 3e(1+i)t )3 .
Solution.
UsingEulers formula
we find

i 2t
1. 2e
= 2 cos 2t + 2i sin 2t.
2. 12 e2t+i(t+) = 21 e2t cos (t + ) 21 e2t i sin (t + ) = 21 e2t cos t + 12 ie2t sin t.

3. ( 3e(1+i)t )3 = 3 3e3(1+i)t = 3 3e3t cos (3t) + 3 3ie3t sin (3t)


In Problems 14.4 - 14.8
(a) Determine the roots of the characteristic equation.
(b) Obtain the general solution as a linear combination of real-valued solutions.
(c) Impose the initial conditions and solve the initial value problem.
Problem 14.4
y 00 + 2y 0 + 2y = 0, y(0) = 3, y 0 (0) = 1.
Solution.
(a) The characteristic equation r2 + 2r + 2 = 0 has roots r1 = 1 i and
r2 = 1 + i.
(b) y(t) = et (c1 cos t + c2 sin t).
(c) The initial conditions and y 0 (t) = et cos t(c2 c1 ) et sin t(c1 + c2 ) lead
to the equations c1 = 3 and c1 + c2 = 1. Solving, we find c1 = 3 and
c2 = 2. Hence, the unique solution to the initial value problem is
y(t) = 3et cos t + 2et sin t
Problem 14.5
2y 00 2y 0 + y = 0, y() = 1, y 0 () = 1.
Solution.
(a) The characteristic equation 2r2 2r + 1 = 0 has roots r1 = 21 (1 i) and

109
r2 = 12 (1 + i).
t
(b) y(t) = e 2 (c1 cos 2t + c2 sin 2t ).
t
t
(c) The initial conditions and y 0 (t) = 21 e 2 cos 2t (c1 + c2 ) + 12 e 2 sin 2t (c1 + c2 )

lead to the equations e 2 c2 = 1 and c2 c1 = 2e 2 . Solving, we find

c1 = 3e 2 and c2 = e 2 . Hence, the unique solution to the initial value


problem is
1
t
t
y(t) = e 2 (t+) (3 cos + sin )
2
2
Problem 14.6

y 00 + 4y 0 + 5y = 0, y( ) = , y 0 ( ) = 2.
2
2
2
Solution.
(a) The characteristic equation r2 + 4r + 5 = 0 has roots r1 = 2 i and
r2 = 2 + i.
(b) y(t) = e2t (c1 cos t + c2 sin t).
(c) The initial conditions and y 0 (t) = e2t cos t(c2 2c1 ) e2t sin t(c1 + 2c2 )
lead to the equations e c2 = 21 and c1 + 2c2 = 2e . Solving, we find c1 = e
and c2 = 21 e . Hence, the unique solution to the initial value problem is
y(t) = e2t (cos t +

1
sin t)
2

Problem 14.7
y 00 + 4 2 y = 0, y(1) = 2, y 0 (1) = 1.
Solution.
(a) The characteristic equation r2 + 4 2 = 0 has roots r1 = 2i and r2 =
2i.
(b) y(t) = c1 cos 2t + c2 sin 2t.
(c) The initial conditions and y 0 (t) = 2c2 cos 2t 2c1 sin 2t lead to the
equations c1 = 2 and 2c2 = 1. Solving, we find c1 = 2 and c2 = (2)1 .
Hence, the unique solution to the initial value problem is
y(t) = 2 cos 2t + (2)1 sin 2t

110

14 CHARACTERISTIC EQUATIONS WITH COMPLEX ROOTS

Problem 14.8
9y 00 + 2 y = 0, y(3) = 2, y 0 (3) = .
Solution.
(a) The characteristic equation 9r2 + 2 = 0 has roots r1 = 3 i and r2 = 3 i.
(b) y(t) = c1 cos 3 t + c2 sin 3 t.
(c) The initial conditions and y 0 (t) = 3 c2 cos 3 t 3 c1 sin 3 t lead to the equations c1 = 2 and 3 c2 = . Solving, we find c1 = 2 and c2 = 3. Hence,
the unique solution to the initial value problem is

y(t) = 2 cos t + 3 sin t


3
3
In Problems 14.9 - 14.10, the function y(t) is a solution of the initial value
problem y 00 + ay 0 + by = 0, y(t0 ) = y0 , y 0 (t0 ) = y00 , where the point t0 is
specified. Determine the constants a, b, y0 , and y00 .
Problem 14.9
y(t) = 2 sin 2t + cos 2t, t0 =

.
4

Solution.
The roots of the characteristic equation are r1,2 = 2i so that the characteristic equation is r2 + 4 = 0. Hence, the corresponding differential equation
is y 00 + 4y = 0. From this, we find a = 0 and b = 4. Now, y0 = y( 4 ) =
2 sin 2 + cos 2 = 2. Finally, y00 = y 0 ( 4 ) = 4 cos 2 2 sin 2 = 2
Problem 14.10

y(t) = et 6 cos 2t et 6 sin 2t, t0 =

.
6

Solution.
The roots of the characteristic equation are r1,2 = 1 2i so that the characteristic equation is r2 2r + 5 = 0. Hence, the corresponding differential equation is y 00 2y 0 + 5y = 0. From this,
we find a = 2 and

b = 5. Now, y0 = y( 6 ) = cos 3 sin 3 = 12 23 . Finally, y00 = y 0 ( 6 ) =


cos 3 sin 3 2 cos 3 2 sin 3 = 12 3 2 3

111
In Problems 14.11 - 14.13, rewrite the function y(t) in the form y(t) =
Ket cos (t ), where 0 < 2. Use this representation to sketch a
graph of the given function, on a domain sufficiently large to display its
main features.
Problem 14.11
y(t) = sin t + cos t.
Solution.

2 + 12 =
1
2. Moreover, cos =
We have
c
=
1
and
c
=
1
so
that
K
=
1
2

c1
2
2

= 2 and sin = 2 . Thus, = 4 and


K
y(t) =



2 cos t
.
4

The graph of y(t) is given below.

Problem 14.12
y(t) = et cos t + 3et sin t.
Solution.

We have c1 = 1 and
3 so that K = 1 + 3 = 2. Moreover, cos =
c2 =
c1
= 12 and sin = 23 . Thus, = 3 and
K


y(t) = 2et cos t
.
3

112

14 CHARACTERISTIC EQUATIONS WITH COMPLEX ROOTS

The graph of y(t) is given below.

Problem 14.13
y(t) = e2t cos 2t e2t sin 2t.
Solution.

We have c1 = 1 and c2 = 1 so that K = 1 + 1 = 2. Moreover, cos =


c1
= 12 and sin = 12 . Thus, = 7
and
K
4


2t
7
.
y(t) = 2e cos 2t
4
The graph of y(t) is given below.

Problem 14.14
Consider the differential equation y 00 + ay 0 + 9y = 0, where a is a real number.

113
Suppose that we know the Wronskian of a fundamental set of solutions of
this differential equation is constant: W (t) = 1 for all real numbers t. Find
the general solution of this differential equation.
Solution.
First we need to find a. Since W 0 (t) = aW (t), a = 0 so that y 00 + 9y = 0.
The characteristic equation is r2 + 9 = 0 and has complex roots r1,2 = 3i.
Thus, the general solution is given by
y(t) = c1 cos 3t + c2 sin 3t
Problem 14.15
Rewrite 2 cos 7t 11 sin 7t in phase-angle form. Give the exact function (so
your answer will involve the inverse tangent function.)
Solution.

We have c1 = 2 and c2 = 11 so that K = 4 + 121 = 125 = 5 5.


so that = arctan 11
Furthermore, tan = 11
. Hence,
2
2

 

11
y(t) = 5 5 cos 7t + arctan
2
Problem 14.16
Find a homogeneous linear ordinary differential equation whose general solution is y(t) = c1 e2t cos (3t) + c2 e2t sin (3t).
Solution.
The roots to the characteristic equation are r1,2 = 2 3i so that the characteristic equation is r2 4r + 13 = 0 and the corresponding differential
equation is
y 00 4y 0 + 13y = 0
Problem 14.17
Rewrite y(t) = 5e(52i)t 3e(5+2i)t , without complex exponents, using sines
and cosines. What ODE of the form ay 00 + by 0 + cy = 0, has y as a solution?
Solution.
Using Eulers formula, we have e(52i)t = e5t (cos 2t i sin 2t) and e(5+2i)t =
e5t (cos 2t + i sin 2t). Thus, y(t) = 2e5t cos 2t 8ie5t sin 2t. The characteristic
roots are r1,2 = 5 2i so that the characteristic equation is r2 10r + 29 and
the corresponding differential equation is
y 00 10y 0 + 29y = 0

114

14 CHARACTERISTIC EQUATIONS WITH COMPLEX ROOTS

Problem 14.18
Consider the function y(t) = 3 cos 2t 4 sin 2t. Find a second order linear
IVP that y satisfies.
Solution.
The roots to the characteristic equation are r1,2 = 2i so that the characteristic equation is r2 + 4 = 0 and the corresponding differential equation
is
y 00 + 4y = 0, y(0) = 3, y 0 (0) = 8
Problem 14.19
An equation of the form
t2 y 00 + ty 0 + y = 0, t > 0
where and are real constants is called an Euler equation. Show that
the substitution x = ln t transforms Euler equation into an equation with
dy dx
= dx
.
constant coefficients. Hint: dy
dt
dt
Solution.
= 1 . But
Since x = ln t, wefind dx
dt  t
dy
d2 y
d2 y
dy
1
t12 dx
+ t12 dx
dx
. Hence,
2 = t2
dx2

dy
dt

dy dx
dx dt

1 dy
.
t dx

Moreover,

d2 y
dt2

0 =t2 y 00 + ty 0 + y
  2



1 dy
dy
1 dy
2
=t

+ t
+ y
t2 dx2 dx
t dx
dy
d2 y
= 2 + ( 1) + y
dx
dx
Problem 14.20
Use the result of the previous problem to solve the differential equation t2 y 00 +
ty 0 + y = 0.
Solution.
Here we have = = 1 so that
d2 y
+y =0
dx2

115
The characteristic equation is r2 + 1 = 0 with complex roots r1,2 = i. The
general solution is
y(x) = c1 cos x + c2 sin x
or
y(t) = c1 cos (ln t) + c2 sin (ln t)

116

14 CHARACTERISTIC EQUATIONS WITH COMPLEX ROOTS

15 Series Solutions of
Differential Equations
Problem 15.1
Identify the singular and ordinary points of the differential equation
y 00 + ty 0 + (t2 + 2)y = 0.
Solution.
Since p(t) = t and q(t) = t2 + 2 are polynomials, they are analytic functions
everywhere. Thus, every real number is an ordinary point
Problem 15.2
Identify the singular and ordinary points of the differential equation
1
(t2 1)y 00 + ty 0 + y = 0.
t
Solution.
t
We have p(t) = t2 1
and q(t) = t(t211) . Thus, the points 1, 1, and 0 are
singular points of the equation, any other real number is an ordinary point
of the equation
Problem 15.3
Identify the singular and ordinary points of the differential equation
(t2 4)y 00 + 3ty 0 + y = 0.
Solution.
We have p(t) = t23t4 and q(t) = t214 . Thus, the singular points are t = 2
and any other point is an ordinary point
117

118

15 SERIES SOLUTIONS OF DIFFERENTIAL EQUATIONS

Problem 15.4
The point t0 = 0 is an ordinary point of the differential equation
(t2 4)y 00 + 3ty 0 + y = 0.
Determine a value of R as stated in Theorem 18.1.
Solution.
The point t0 = 0 is an ordinary point. By the previous exercise, both p(t)
and q(t) are analytic in 2 < t < 2 so that a choice of R is R = 2
Problem 15.5
Find the series solution near the ordinary point 0 to the differential equation
y 00 + ty 0 + (t2 + 2)y = 0.
Solution.
There are no singular points. Thus, the series solution converges for every
value of t. The solution is of the form
y(t) =

an tn .

n=0

Differentiating this series twice we find


P
P
n2
n1
.
and y 00 (t) =
y 0 (t) =
n=2 n(n 1)an t
n=1 nan t
Substituting these series in the given equation we find

n2

n(n 1)an t

n=2

+t

n1

nan t

+t

n=1

X
n=2

n(n 1)an tn2 +

X
n=1

an t + 2

n=0

nan tn +

X
n=0

an tn+2 +

an tn =0

n=0

2an tn =0

n=0

X
X
X
X
n
n
n
(n + 2)(n + 1)an+2 t +
nan t +
an2 t +
2an tn =0.
n=0

n=1

n=2

n=0

We shift the index of summation in the first series by 2, replacing n with


n + 2 and using the initial value n = 0. We shift the index of summation in
the third series by 2, replacing n by n 2 and using the initial value n = 2.

119
Since we want to express everything in only one summation sign, we have to
start the summation at n = 2 in every series

(n + 2)(n + 1)an+2 tn +

n=0

nan tn +

n=1

an2 tn +

n=2

2an tn =

n=0

X
X
n
2a2 + 6a3 t +
(n + 2)(n + 1)an+2 t + a1 t +
nan tn +
n=2

an2 tn + 2a0 + 2a1 t +

n=2

n=2

2an tn =0

n=2

or

(2a0 +2a2 )+(6a3 +3a1 )t+

[(n + 2)(n + 1)an+2 + (n + 2)an + an2 ] tn = 0.

n=2

Equating all the coefficients to zero we find

2a0 + 2a2 =0
3a1 + 6a3 =0
(n + 1)(n + 2)an+2 + (n + 2)an + an2 =0.

Thus,

a2 = a0
a1
a3 =
2
(n + 2)an + an2
an+2 =
, n 2.
(n + 1)(n + 2)

120

15 SERIES SOLUTIONS OF DIFFERENTIAL EQUATIONS

This last condition is called a recurrence formula, we can express each


an+2 in terms of a previous coefficient an . From the above, we can see that
a2 = a0
a1
a3 =
2
1
a4 = a0
4
3
a5 = a1
40
1
a6 = a0
60
1
a1 .
a7 =
1680
Notice that each even coefficient is expressed in terms of a0 and each odd
coefficient is expressed in terms of a1 . Then, the general solution is:




1 6
3 5
1 7
1 3
1 4
2
t +
y(t) = a0 1 t + t t + +a1 t t + t
4
60
2
40
1680
Problem 15.6
Find the series solution near the ordinary point 0 to the differential equation
y 00 ty 0 t2 y = 0.
Solution.
There are no singular points. Thus, the series solution converges for every
value of t. The solution is of the form
y(t) =

an tn .

n=0

Differentiating this series twice we find


y 0 (t) =

n=1

nan tn1 and y 00 (t) =

n=2

n(n 1)an tn2 .

121
Substituting these series in the given equation we find

n2

n(n 1)an t

n=2

nan t

n=1

n(n 1)an tn2

n=2

n1

nan tn

n=1

an tn =0

n=0

an tn+2 =0

n=0

X
X
X
n
n
(n + 2)(n + 1)an+2 t
nan t
an2 tn =0.
n=0

n=1

n=2

We shift the index of summation in the third series by 2, replacing n by


n 2 and using the initial value n = 2.
Since we want to express everything in only one summation sign, we have to
start the summation at n = 2 in every series

X
X
X
n
n
(n + 2)(n + 1)an+2 t
nan t
an2 tn =
n=0

2a2 + 6a3 t +

(n + 2)(n + 1)an+2 tn a1 t

n=2

n=1

n=2

nan tn

n=2

2a2 + (6a3 a1 )t +

an2 tn =

n=2

[(n + 1)(n + 2)an+2 nan an2 ] tn =0.

n=2

Equating all the coefficients to zero we find


2a2 =0
6a3 a1 =0
(n + 1)(n + 2)an+2 nan an2 =0.
Thus,
a2 =0
a1
a3 =
6
nan + an2
an+2 =
, n 2.
(n + 2)(n + 1)

122

15 SERIES SOLUTIONS OF DIFFERENTIAL EQUATIONS

This last condition is called a recurrence formula, we can express each


an+2 in terms of a previous coefficient an . From the above, we can see that
a2 =0
a1
a3 =
6
1
a4 = a0
12
3
a5 = a1
40
1
a6 = a0
90
13
a1 .
a7 =
1008
Notice that each even coefficient is expressed in terms of a0 and each odd
coefficient is expressed in terms of a1 . Then, the general solution is:




1 6
3 5
13 7
1 3
1 4
t +
y(t) = a0 1 + t + t + + a1 t + t + t +
12
90
6
40
1008
Problem 15.7
Find the series solution near the ordinary point 0 to the differential equation
(t2 + 1)y 00 y 0 + y = 0.
Solution.
There are no singular points. Thus, the series solution converges for every
value of t. The solution is of the form
y(t) =

an tn .

n=0

Differentiating this series twice we find


y 0 (t) =

n=1

nan tn1 and y 00 (t) =

n=2

n(n 1)an tn2 .

123
Substituting these series in the given equation we find
t2

n(n 1)an tn2 +

n=2

n(n 1)an tn +

n=2

n(n 1)an tn2

n=2

nan tn1 +

n=2

n=1

n=0

n(n 1)an tn2

n=2

n(n 1)an tn +

nan tn1 +

n=1

n=0

n=0

(n + 2)(n + 1)an+2 tn

(n + 1)an+1 tn +

n=0

an tn =0
an tn =0
an tn =0.

n=0

Since we want to express everything in only one summation sign, we have to


start the summation at n = 2 in every series

X
X
X
n
n
n(n 1)an t +
(n + 2)(n + 1)an+2 t
(n + 1)an+1 t +
an tn =

n=2

n=0

n=0

n=0

X
n
n(n 1)an t + 2a2 + 6a3 t +
(n + 2)(n + 1)an+2 tn

n=2

n=2

a1 2a2 t

n=2

n=2

(n + 1)an+1 tn + a0 + a1 t +

an tn =0.

This can be written in the form

X


(2a2 a1 +a0 )+(6a3 2a2 +a1 )t+
(n + 2)(n + 1)an+2 + (n2 n + 1)an (n + 1)an+1 tn = 0.
n=2

Equating all the coefficients to zero we find


2a2 a1 + a0 =0
6a3 2a2 + a1 =0
2
(n + 1)(n + 2)an+2 + (n n + 1)an (n + 1)an+1 =0.
Thus,
a1 a0
2
2a2 a1
a0
a3 =
=
6
6
(n + 1)an+1 (n2 n + 1)an
an+2 =
, n 2.
(n + 1)(n + 2)
a2 =

124

15 SERIES SOLUTIONS OF DIFFERENTIAL EQUATIONS

This last condition is called a recurrence formula, we can express each


an+2 in terms of a previous coefficient an . From the above, we can see that
a1 a0
2
a0
=
6
1
= (2a0 3a1 )
24
1
= (3a0 a1 )
40
1
= a0
90
1
(36a1 17a0 )
=
720

a2 =
a3
a4
a5
a6
a7

The general solution is:






1 2 1 3
1 4
3 5
1 5
1 2 1 4
y(t) = a0 1 t t + t + t + +a1 t + t t t +
2
6
12
40
2
8
40
Problem 15.8
Write the following as a single power series:
X
X
n2 an (t 2)n .
nan (t 2)n+1 +
n=2

n=0

Solution.
We have
X
X
X
X
n2 an (t 2)n
nan (t 2)n+1 +
n2 an (t 2)n =
(n 1)an1 (t 2)n +
n=0

n=2

n=2

n=1

(n 1)an1 (t 2) +

n=2

n2 an (t 2)n

n=2
2

[(n 1)an1 + n an ](t 2)n

n=2

Problem 15.9
Find the series solution near the ordinary point 0 to the differential equation
y 00 + ty 0 + y = 0.

125
Solution.
We seek solution of the form
y(t) =

an tn .

n=0

Differentiating this series twice we find


y 0 (t) =

n=1

nan tn1 and y 00 (t) =

n=2

n(n 1)an tn2 .

Substituting these series in the given equation we find

n2

n(n 1)an t

+t

n=2

nan t

n=1

n(n 1)an tn2 +

n=2

(n + 1)(n + 2)an+2 tn +

n1

nan tn +

n=0

n=1

n=0

nan tn +

n=1

n=0

an tn =0
an tn =0
an tn =0.

n=0

Since we want to express everything in only one summation sign, we have to


start the summation at n = 1 in every series
(2a2 + a0 ) +

[(n + 2)(n + 1)an+2 + (n + 1)an ] tn = 0.

n=1

Equating all the coefficients to zero we find


2a2 + a0 =0
(n + 1)(n + 2)an+2 + (n + 1)an =0.
Thus,
a0
2
an
=
, n 1.
n+2

a2 =
an+2

126

15 SERIES SOLUTIONS OF DIFFERENTIAL EQUATIONS

This last condition is called a recurrence formula, we can express each


an+2 in terms of a previous coefficient an . From the above, we can see that
a0
a2 =
2
a1
a3 =
3
a0
a4 =
8
a1
a5 =
15
a0
a6 =
48
a1
.
a7 =
105
Notice that each even coefficient is expressed in terms of a0 and each odd
coefficient is expressed in terms of a1 . Then, the general solution is:




1 6
1 5
1 7
1 2 1 4
1 3
y(t) = a0 1 t + t t + +a1 t t + t
t +
2
8
48
3
15
105
Problem 15.10
Find the series solution near the ordinary point 1 to the differential equation
(1 t)2 y 00 2y = 0.
Solution.
We seek solution of the form
y(t) =

an tn .

n=0

Differentiating this series twice we find


P
P
n1
n2
y 0 (t) =
and y 00 (t) =
.
n=1 nan t
n=2 n(n 1)an t
Substituting these series in the given equation we find

X
X
2
n2
(1 2t + t )
n(n 1)an t
2
an tn =0

n(n 1)an tn 2

n=2

X
n=2

n(n 1)an tn 2

X
n=2

X
n=1

n(n 1)an tn1 +

n=2

n=0

n(n 1)an tn2 2

n=2

n
n(n + 1)an+1 tn + sum
n=0 (n + 1)(n + 2)an+2 t 2

n=0

X
n=0

an tn =0
an tn =0.

127
Since we want to express everything in only one summation sign, we have to
start the summation at n = 2 in every series

X


(2a2 2a0 )+(6a3 2a1 4a2 )t+
(n + 2)(n + 1)an+2 2n(n + 1)an+1 + (n2 n 2)an tn = 0.
n=2

Equating all the coefficients to zero we find


2a2 2a0 =0
6a3 2a1 4a2 =0
(n + 2)(n + 1)an+2 2n(n + 1)an+1 + (n2 n 2)an =0.
Thus,
a2 =a0
a1 + 2a0
a3 =
3
2nan+1 (n 2)an
, n 2.
an+2 =
n+2
From the above, we can see that
a2 =a0
a1 + 2a0
a3 =
3
a4 =a3
a5 =a3
a6 =a3
a7 =a3 .
The general solution is:




2 3 2 4 2 5
1 3 1 4 1 5
2
y(t) = a0 1 + t + t + t + t + +a1 t + t + t + t +
3
3
3
3
3
3

128

15 SERIES SOLUTIONS OF DIFFERENTIAL EQUATIONS

16 The Structure of the


General Solution of Linear
Nonhomogeneous Equations
In Problems 16.1- 16.6, answer the following three questions.
(a) Verify that the given function, yp (t), is a particular solution of the differential equation.
(b) Determine the general solution,yh , of the homogeneous equation.
(c) Find the general solution to the differential equation and impose the
initial conditions to obtain the unique solution of the initial value problem.
Problem 16.1
1
y 00 2y 0 3y = e2t , y(0) = 1, y 0 (0) = 0, yp (t) = e2t .
3
Solution.
(a) yp0 = 23 e2t , yp00 = 34 e2t .
4
4
yp00 2yp0 3yp = e2t + e2t + e2t
3
3
2t
=e .
(b) The associated characteristic equation r2 2r 3 = 0 has roots r1 = 1
and r2 = 3. Hence, the general solution to the homogeneous differential
equation is
yh (t) = c1 et + c2 e3t .
(c) The general solution to the differential equation is y(t) = c1 et + c2 e3t
1 2t
e . The derivative of this function is given by y 0 (t) = c1 et + 3c2 e3t 32 e2t .
3
129

13016 THE STRUCTURE OF THE GENERAL SOLUTION OF LINEAR NONHOMOGENEO


The condition y(0) = 1 leads to c1 + c2 = 34 . The condition y 0 (0) = 0 leads to
c1 + 3c2 = 32 . Solving for c1 and c2 , we find c1 = 65 and c2 = 21 . The unique
solution is given by
5
1
1
y(t) = et + e3t e2t
6
2
3
Problem 16.2
y 00 y 0 2y = 10, y(1) = 0, y 0 (1) = 1, yp (t) = 5.
Solution.
(a) yp0 = yp00 = 0.
yp00 yp0 2yp = 0 0 2(5) = 10.
(b) The associated characteristic equation r2 r2 = 0 has roots r1 = 1 and
r2 = 2. Hence, the general solution to the homogeneous differential equation
is
yh (t) = c1 et + c2 e2t .
(c) The general solution to the differential equation is y(t) = c1 et + c2 e2t
5. The derivative of this function is given by y 0 (t) = c1 et + 2c2 e2t . The
condition y(1) = 0 leads to c1 e + c2 e2 = 5. The condition y 0 (1) = 1 leads
to c1 e + 2c2 e2 = 1. Solving for c1 and c2 , we find c1 = 3e and c2 = 2e2 . The
unique solution is given by
y(t) = 3e(t+1) + 2e2t+2 5
Problem 16.3
y 00 + y 0 = 2et , y(0) = 2, y 0 (0) = 2, yp (t) = 2tet .
Solution.
(a) yp0 = 2et + 2tet , yp00 = 4et 2tet .
yp00 + yp0 =4et 2tet 2et + 2tet
=2et .
(b) The associated characteristic equation r2 + r = 0 has roots r1 = 0 and
r2 = 1. Hence, the general solution to the homogeneous differential equation
is
yh (t) = c1 + c2 et .

131
(c) The general solution to the differential equation is y(t) = c1 +c2 et 2tet .
The derivative of this function is given by y 0 (t) = c2 et 2et + 2tet . The
condition y(0) = 2 leads to c1 + c2 = 2. The condition y 0 (0) = 2 leads to
c2 2 = 2. Solving for c1 and c2 , we find c1 = 6 and c2 = 4. The unique
solution is given by
y(t) = 6 4et 2tet
Problem 16.4
y 00 + 4y = 10et , y() = 2, y 0 () = 0, yp (t) = 2et .
Solution.
(a) yp0 = yp00 = 2et .
yp00 + 4yp0 =2et + 8et
=10et .
(b) The associated characteristic equation r2 + 4 = 0 has roots r1 = 2i and
r2 = 2i. Hence, the general solution to the homogeneous differential equation
is
yh (t) = c1 cos 2t + c2 sin 2t.
(c) The general solution to the differential equation is y(t) = c1 cos 2t +
c2 sin 2t+2et . The derivative of this function is given by y 0 (t) = 2c1 sin 2t+
2c2 cos 2t + 2et . The condition y() = 2 leads to c1 + 2 = 2. The condition
y 0 () = 0 leads to 2c2 + 2 = 0. Solving for c1 and c2 , we find c1 = 0 and
c2 = 1. The unique solution is given by
y(t) = sin 2t + 2et
Problem 16.5

y 00 2y 0 + 2y = 5 sin t, y( ) = 1, y 0 ( ) = 0, yp (t) = 2 cos t + sin t.


2
2
Solution.
(a) yp0 = 2 sin t + cos t, yp00 = 2 cos t sin t.
yp00 2yp0 + 2yp = 2 cos t sin t + 4 sin t 2 cos t + 4 cos t + 2 sin t
=5 sin t.

13216 THE STRUCTURE OF THE GENERAL SOLUTION OF LINEAR NONHOMOGENEO


(b) The associated characteristic equation r2 2r + 2 = 0 has roots r1 = 1 i
and r2 = 1 + i. Hence, the general solution to the homogeneous differential
equation is
yh (t) = et (c1 cos t + c2 sin t).
(c) The general solution to the differential equation is y(t) = et (c1 cos t +
c2 sin t) + 2 cos t + sin t. The derivative of this function is given by y 0 (t) =
et cos t(c1 + c2 ) + et sin t(c1 + c2 ) 2 sin t + cos t. The condition y( 2 ) = 1

leads to e 2 c2 + 1 = 1. The condition y 0 ( 2 ) = 0 leads to e 2 c1 2 = 0.

Solving for c1 and c2 we find c1 = 2e 2 and c2 = 0. The unique solution is


given by

y(t) = 2et 2 cos t + 2 cos t + sin t


Problem 16.6
y 00 2y 0 + y = t2 + 4 + 2 sin t, y(0) = 1, y 0 (0) = 3, yp (t) = t2 + 4t + 10 + cos t.
Solution.
(a) yp0 = 2t + 4 sin t, yp00 = 2 cos t.
yp00 2yp0 + yp =2 cos t 4t 8 + 2 sin t + t2 + 4t + 10 + cos t
=t2 + 4 + 2 sin t.
(b) The associated characteristic equation r2 2r + 1 = 0 has roots r1 =
r2 = 1. Hence, the general solution to the homogeneous differential equation
is
yh (t) = c1 et + c2 tet .
(c) The general solution to the differential equation is y(t) = c1 et + c2 tet +
t2 + 4t + 10 + cos t. The derivative of this function is given by y 0 (t) = c1 et +
c2 et + c2 tet + 2t + 4 sin t. The condition y(0) = 1 leads to c1 + 10 + 1 = 1.
The condition y 0 (0) = 3 leads to c1 + c2 + 4 = 3. Solving for c1 and c2 , we
find c1 = 10 and c2 = 9. The unique solution is given by
y(t) = 10et + 9tet + t2 + 4t + 10 + cos t
The functions u1 , u2 , and u3 are solutions to the following differential equations
u00 + p(t)u0 + q(t)u =2et + 1
u00 + p(t)u0 + q(t)u =2et t 1
u00 + p(t)u0 + q(t)u =3t.

133
In Problems 16.7 - 16.8, use the functions u1 , u2 and u3 to construct a particular solution of the differential equation.
Problem 16.7
1
u00 + p(t)u0 + q(t)u = et + 2t + .
2
Solution.
The right-hand side of the given equation can be written as et + 2t + 12 =
1
(2et +1)+ 23 (3t) so that by Theorem 16.2, the function u(t) = 12 u1 (t)+ 32 u3 (t)
2
is the required particular solution
Problem 16.8
u00 + p(t)u0 + q(t)u =

et + et
.
2

Solution.
t
t
= 14 (2et +
The right-hand side of the given equation can be written as e +e
2
1
(3t) so that by Theorem 16.2, the function u(t) =
1) + 14 (2et t 1) + 12
1
1
1
u (t) + 4 u2 (t) + 12 u3 (t) is the required particular solution
4 1
In Problems 16.9 - 16.11, determine the function g(t).
Problem 16.9
y 00 2y 0 = g(t), yp (t) = 3t +
Solution.
We have yp0 = 3 +

2 t

and yp00 =

1
3

4t 2

t, t > 0.

. Thus,

g(t) =yp00 2yp0


1
1 3
= t 2 6 t 2
4
Problem 16.10
y 00 + y 0 = g(t), yp (t) = ln (1 + t), t > 1.

13416 THE STRUCTURE OF THE GENERAL SOLUTION OF LINEAR NONHOMOGENEO


Solution.
We have yp0 =

1
1+t

and yp00 = (1 + t)2 . Thus,


g(t) =yp00 + yp0
1
1
=
+
2
(1 + t)
1+t

Problem 16.11
y 00 + 2y 0 + y = g(t), yp (t) = t 2.
Solution.
We have yp0 = 1 and yp00 = 0. Thus,
g(t) =yp00 + 2yp0 + yp
=0 + 2 + t 2
=t
In Problems 16.12 - 16.13, the general solution of the nonhomogeneous differential equation y 00 + y 0 + y = g(t) is given, where c1 and c2 are arbitrary
constants. Determine the constants and and the function g(t).
Problem 16.12
y(t) = c1 et + c2 tet + t2 et .
Solution.
From the given general solution we see that the roots of the characteristic
equation are r1 = r2 = 1. Thus, the characteristic equation is (r 1)(r 1) =
r2 2r +1 = 0. The associated differential equation is y 00 2y 0 +y = 0. Hence,
= 2 and = 1. Now,
g(t) =yp00 2yp0 + yp
=2et + 4tet + t2 et 4tet 2t2 et + t2 et
=2et
Problem 16.13
y(t) = c1 sin 2t + c2 cos 2t 1 + sin t.

135
Solution.
From the given general solution we see that the roots of the characteristic
equation are r1 = 2i and r2 = 2i. Thus, the characteristic equation is
(r2i)(r+2i) = r2 +4 = 0. The associated differential equation is y 00 +4y = 0.
Hence, = 0 and = 4. Now,
g(t) =yp00 + 4yp
= sin t + 4 sin t 4
=3 sin t 4
Problem 16.14
t
Given that the function e5 satisfies the differential equation y 00 + 4y = et ,
write a general solution of the differential equation y 00 + 4y = et .
Solution.
First, we find yh . The characteristic equation r2 + 4 = 0 has the roots
r1,2 = 2i. Thus, yh (t) = c1 cos 2t + c2 sin 2t. The general solution to the
nonhomogeneous equation is
y(t) = c1 cos 2t + c2 sin 2t +

et
5

Problem 16.15
Find the general solution to the differential equation
y (4) + 9y 00 = 24 + 108t2
given a particular solution yp (t) = cos 3t + sin 3t + t4 .
Solution.
Let z = y 00 . Then the given equation reduces to a second order differential
equation
z 00 + 9z = 24 + 108t2 .
The characteristic equation is r2 + 9 = 0 so that the roots are r1,2 = 3i.
Thus, zh (t) = c1 cos 3t + c2 sin 3t. Integrating this function twice we find
yh (t) = c1 cos 3t + c2 sin 3t + c3 t + c4 . Hence, the general solution to the given
differential equation is
y(t) = c1 cos 3t + c2 sin 3t + c3 t + c4 + t4

13616 THE STRUCTURE OF THE GENERAL SOLUTION OF LINEAR NONHOMOGENEO

17 The Method of Variation of


Parameters
Problem 17.1
Solve y 00 + y = sec t by variation of parameters.
Solution.
The characteristic equation r2 + 1 = 0 has roots r = i and
yh (t) = c1 cos t + c2 sin t.
Also, y1 (t) = cos t and y2 (t) = sin t so that W (t) = cos2 t + sin2 t = 1. Now,
Z
u1 =

Z
sin t sec tdt =

d(cos t)
= ln | cos t|
cos t

and
Z
u2 =

Z
cos t sec tdt =

dt = t.

Hence, the particular solution is given by


yp (t) = ln | cos t| cos t + t sin t
and the general solution is
y(t) = c1 cos t + c2 sin t + ln | cos t| cos t + t sin t
Problem 17.2
Solve y 00 y = et by variation of parameters.
137

138

17 THE METHOD OF VARIATION OF PARAMETERS

Solution.
The characteristic equation r2 1 = 0 for y 00 y = 0 has roots r = 1. The
homogeneous solution is
yh (t) = c1 et + c2 et .
The homogeneous solution yh (t) = c1 et + c2 et found above implies y1 =
et , y2 = et is a suitable independent pair of solutions. Their Wronskian is
W = 2. The variation of parameters formula applies:
Z t
Z t
e t
e t
t
t
e dt e
e dt.
yp (t) = e
2
2
Integration, followed by setting all constants of integration to zero, gives
t
t
yp (t) = te2 e4 .
The general solution is
1
y(t) = c1 et + c2 et + tet
2
Problem 17.3
Solve the following 2nd order equation using the variation of parameters
method:
y 00 + 4y = t2 + 8 cos 2t.
Solution.
The characterisitc equation r2 + 4 = 0 has roots r = 2i so that yh (t) =
c1 cos 2t + c2 sin 2t. Hence, y1 (t) = cos 2t, y2 (t) = sin 2t, and W (t) = 2. Thus,
Z
Z
cos 2t(t2 + 8 cos 2t)
sin 2t(t2 + 8 cos 2t)
dt + sin 2t
dt
yp = cos 2t
2
2
1
1
1
= cos 2t( t sin 2t + cos 2t t2 cos 2t cos2 2t)
4
8
4
1
1
1 2
1
+ sin 2t( t cos 2t sin 2t + t sin 2t + 2t + sin 4t)
4
8
4
2
1 1 2
1
= + t + cos2 2t cos 2t + 2t sin 2t + sin 4t sin 2t.
8 4
2
The general solution is
y(t) = c1 cos 2t + c2 sin 2t

1 1 2
+ t + 2t sin 2t
8 4

139
Problem 17.4
Find a particular solution by the variation of parameters to the equation
y 00 + 2y 0 + y = et ln t.
Solution.
The characteristic equation
r2 + 2r + 1 = 0
has roots r1 = r2 = 1, so the fundamental solutions of the reduced equation
are
y1 (t) = et , y2 (t) = tet .
Compute the Wronskian.
t
e
tet
W (t) =
et et tet

=et (et tet ) + et tet


=e2t te2t + te2t
=e2t .
Compute u1 (t).
Z
u1 (t) =
Z
=
Z
=
=

t2
2

y2 (t)g(t)
dt
W (t)
tet et ln t
dt
e2t
Z 2
t2
t 1
t ln tdt = ln t +
dt
2
2 t
t2
ln t + .
4

Compute u2 (t).
Z

y1 (t)g(t)
dt
W (t)
Z t t
e e ln t
=
dt
e2t
Z
Z
1
= ln tdt = t ln t t dt
t
=t ln t t.

u2 (t) =

140

17 THE METHOD OF VARIATION OF PARAMETERS

Note.
We used integration by parts to compute the integrals
R
ln tdt.
The particular solution to our complete equation is

t ln tdt and

yp (t) =u1 (t)y1 (t) + u2 (t)y2 (t)


 2

t
t2 t
= ln t +
e + (t ln t t)tet
2
4
t2
3t2 t
= ln tet
e
2
4
t2
3t2 t
=( ln t
)e
2
4
Problem 17.5
Solve the following initial value problem by using variation of parameters:
y 00 + 2y 0 3y = tet , y(0) =

1
59
, y 0 (0) = .
64
64

Solution.
From the characteristic equation, we obtain y1 (t) = et , y2 (t) = e3t and
W (t) = 4e2t . Integration then yields
Z 3t t
t2
e te
dt
=
u1 (t) =
4e2t
8
Z
t t
e te
1 4t e4t
u2 (t) =
dt = te +
.
4e2t
16
64
Thus. yp (t) =

et
(8t2
64

4t + 1) and the general solution is


y(t) = c1 et + c2 e3t +

t2 t
1
e tet .
8
16

Initial conditions:
1
64
4
59
y 0 (0) =c1 3c2
= .
64
64
y(0) =c1 + c2 =

These are satisfied by c1 =


value problem is
y=

15
64

and c2 = 14 . Finally the solution to the initial

et 2
1
(8t 4t + 15) e3t
64
4

141
Problem 17.6

(a) Verify that {e t , e t } is a fundamental set for the equation


4ty 00 + 2y 0 y = 0
on the interval (0, ). You may assume that the given functions are solutions
to the equation.
(b) Use the method of variation of parameters to find one solution to the
equation

4ty 00 + 2y 0 y = 4 te t .
Solution.

(a) Usually
the first thing to do would be to check that y1 (t) = e t and

y2 (t) = e t really are solutions to the equation. However, the question says
that this can be assumed and so we move on to the next step, which is to
check that the Wronskian of the two solutions is non-zero on (0, ). We have
y10 =
and so

e
t

1
e
and y20 = 2
t

1
1
1
W (t) = y1 y20 y10 y2 = = .
2 t 2 t
t

This is indeed non-zero and so {e t , e t } is a fundamental set for the homogeneous equation.
(b) The variation of parameters formula says that
Z
Z
y1 g
y2 g
dt + y2
dt
y = y1
W (t)
W (t)
is a solution to the nonhomogeneous equation in the form y 00 + py 0 + qy = g.
To get the
right g, we have to divide the equation through by 4t and so

1
t

g = t e . Thus
Z et ( 1 )et
Z et ( 1 )et

t
t

dt
y =e t
dt + e t
1/ t
1/ t
Z
Z

t
t
=e
dt e
e2 t dt
Z

t
t
=te e
e2 t dt.

142

17 THE METHOD OF VARIATION OF PARAMETERS

To evaluate the integral, we substitute u = 2 t so that dt = 12 udu. We get


Z
Z

1
1
2 t
e dt =
ueu du = (u 1)eu = ( t 1/2)e2 t .
2
2
Thus
y = (t

t + 1/2)e

is one solution to the equation. You might notice that the 1/2 can be dropped
(because (1/2)e t is a solution to the homogeneous equation) so that

y = (t t)e t
would also work
Problem 17.7
Use the method of variation of parameters to find the general solution to the
equation
y 00 + y = sin t.
Solution.
The characteristic equation r2 + 1 = 0 has roots r = i so that the solution
to the homogeneous equation is yh (t) = c1 cos t + c2 sin t. The Wronskian
. Hence, u1 (t) =
is W (cos t, sin t) = 1. Now u01 (t) = sin2 t = cos (2t)1
2
1 1
0
(
sin(2t)

t).
Similarly,
u
(t)
=
sin
t
cos
t.
Hence,
u
(t)
= 21 sin2 t. So
2
2
2 2
yp (t) = 12 t cos t + 12 sin t. The general solution is given by
1
y(t) = c1 cos t + c2 sin t t cos t
2
Problem 17.8
Consider the differential equation
t2 y 00 + 3ty 0 3y = 0, t > 0.
(a) Determine r so that y = tr is a solution.
(b) Use (a) to find a fundamental set of solutions.
(c) Use the method of variation of parameters for finding a particular solution
to
1
t2 y 00 + 3ty 0 3y = 3 , t > 0.
t

143
Solution.
(a) Inserting y, y 0 , and y 00 into the equation we find r2 + 2r 3 = 0. Solving
for r to obtain r1 = 1 and r2 = 3.
(b) Let y1 (t) = t and y2 (t) = t3 . Since


3
t

t
3

W (t) =
4 = 4t
1 3t
{y1 , y2 } is a fundamental set of solutions for t > 0.
(c) Recall that the variation of parameters formula states that if y1 and y2
form a fundamental solution set for y 00 + p(t)y 0 + q(t)y = 0, then yp (t) =
u1 (t)y1 (t) + u2 (t)y2 (t) is a particular solution to the equation y 00 + p(t)y 0 +
q(t)y = g(t), where
Z 3 5
1
t t
dt = t4
u1 (t) =
3
4t
16
Z
5
1
tt
dt = ln t.
u2 (t) =
3
4t
4
Thus,
yp (t) =

1 3 1 3
t t ln t
16
4

Problem 17.9
Use the method of variation of parameters to find the general solution to the
differential equations
y 00 + y = sin2 t.
Solution.
The characterisitc equation r2 + 1 = 0 has roots r = i so that y1 (t) =
cos t, y2 (t) = sin t, and W (t) = 1. Hence,
Z
Z
1
2
u1 (t) = sin t sin tdt = (1 cos2 t)d(cos t) = cos t cos3 t
3
Z
1
u2 (t) = cos t sin2 tdt = sin3 t.
3
Thus,
yp (t) = cos2 t

1
1
cos4 t + sin4 t
3
3

and
y(t) = c1 cos t + c2 sin t + cos2 t

1
1
cos4 t + sin4 t
3
3

144

17 THE METHOD OF VARIATION OF PARAMETERS

18 The Laplace Transform:


Basic Definitions and Results
Problem 18.1
R
Determine whether the integral 0
verges, give its value.

1
dt
1+t2

converges. If the integral con-

Solution.
We have

Z
0

1
dt = lim [arctan t]A
0
2
A
0 1+t

= lim arctan A = .
A
2

1
dt = lim
A
1 + t2

So the integral is convergent


Problem 18.2
R
Determine whether the integral 0
verges, give its value.
Solution.
We have
Z
0

t
dt
1+t2

converges. If the integral con-

Z A


t
1
2t
1
2 A
dt
=
lim
dt
=
lim
ln
(1
+
t
)
0
1 + t2
2 A 0 1 + t2
2 A
1
= lim ln (1 + A2 ) = .
2 A

Hence, the integral is divergent


145

14618 THE LAPLACE TRANSFORM: BASIC DEFINITIONS AND RESULTS


Problem 18.3
R
Determine whether the integral 0 et cos (et )dt converges. If the integral
converges, give its value.
Solution.
Using the substitution u = et we find
Z

eA

cos udu

cos (e )dt = lim

1
A

= lim [ sin u]e1


A

= lim [sin 1 sin (eA )]


A

= sin 1.
Hence, the integral is convergent
Problem 18.4
Using the definition, find L[e3t ], if it exists. If the Laplace transform exists
then find the domain of F (s).
Solution.
We have
A

3t

3t st

L[e ] = lim

e e

= lim


= lim

t(3s)

e
3s

dt = lim

et(3s) dt

A

A(3s)

1
e

3s
3s

1
, s>3
s3

Problem 18.5
Using the definition, find L[t 5], if it exists. If the Laplace transform exists
then find the domain of F (s).

147
Solution.
Using integration by parts we find
(
)

Z A
Z A
st A
(t 5)e
1
(t 5)est dt = lim
L[t 5] = lim
est dt
+
A
A 0
s
s 0
0
(
)


A
est
(A 5)esA + 5

= lim
A
s
s2 0
1
5
, s>0
2
s
s
Problem 18.6
2
Using the definition, find L[e(t1) ], if it exists. If the Laplace transform
exists then find the domain of F (s).
=

Solution.
We have

(t1)2 st

e(t1) st dt.
dt =
0

 0
1
+
= , for any fixed s
Since limt (t 1)2 st = limt t2 1 (2+s)
2
t
t
e

we can choose a positive C such thatR (t 1)2 st 0 Rfor t C. In this case,


2
2

e(t1) st 1 and this implies that 0 e(t1) st dt C dt. The integral on


the right is divergent so that the integral on the left is also divergent by the
2
comparison theorem of improper integrals. Hence, f (t) = e(t1) does not
have a Laplace transform
Problem 18.7
Using the definition, find L[(t 2)2 ], if it exists. If the Laplace transform
exists then find the domain of F (s).
Solution.
We have
Z

L[(t 2) ] = lim

(t 2)2 est dt.

Using integration by parts with u0 = est and v = (t 2)2 we find



T
Z T
Z
(t 2)2 est
2 T
2 st
(t 2) e dt =
+
(t 2)est dt
s
s 0
0
0
Z
2 sT
4 (T 2) e
2 T
=
+
(t 2)est dt.
s
s
s 0

14818 THE LAPLACE TRANSFORM: BASIC DEFINITIONS AND RESULTS


Thus,
Z

2 st

(t 2) e

lim

0
0

4 2
dt = + lim
s s T

(t 2)est dt.

st

Using by parts with u = e


and v = t 2 we find

T
Z T
(t 2)est
1 st
st
(t 2)e dt =
2e
.
s
s
0
0
Letting T in the above expression we find
Z T
1
2
lim
(t 2)est dt = + 2 , s > 0.
T 0
s s
Hence,
4 2
F (s) = +
s s

1
2
+ 2
s s


=

4
2
4
2 + 3, s > 0
s s
s

Problem 18.8
Using the definition, find L[f (t)], if it exists. If the Laplace transform exists
then find the domain of F (s).

0,
0t<1
f (t) =
t 1,
t 1.
Solution.
We have
Z
L[f (t)] = lim

T
0

(t 1)est dt.

1
st

Using integration by parts with u = e


and v = t 1 we find

T
Z T
(t 1)est
1 st
es
st
= 2 , s>0
lim
(t 1)e dt = lim
2e
T 1
T
s
s
s
1
Problem 18.9
Using the definition, find L[f (t)], if it exists. If the Laplace transform exists
then find the domain of F (s).

0t<1
0,
t 1, 1 t < 2
f (t) =

0,
t 2.

149
Solution.
We have

2
Z 2
(t 1)est
1 st
e2s 1 s 2s
st
(t1)e dt =
L[f (t)] =
2e
=
+ 2 (e e ), s 6= 0
s
s
s
s
1
1
Problem 18.10
Let n be a positive integer. Using integration by parts establish the reduction
formula
Z
Z
tn est n
n st
t e dt =
+
tn1 est dt, s > 0.
s
s
Solution.
st
Let u0 = est and v = tn . Then u = e s and v 0 = ntn1 . Hence,
Z

n st

t e

tn est n
dt =
+
s
s

tn1 est dt, s > 0

Problem 18.11
For s > 0 and n a positive integer evaluate the limits
(a) limt0 tn est

(b) limt tn est .

Solution.
n
(a) limt0 tn est = limt0 etst = 10 = 0.
(b) Using LHopitals rule repeatedly we find
lim tn est = = lim

n!

t sn est

=0

Problem 18.12
(a) Use the previous two problems to derive the reduction formula for the
Laplace transform of f (t) = tn ,
L[tn ] =

n n1
L[t ], s > 0.
s

(b) Calculate L[tk ], for k = 1, 2, 3, 4, 5.


(c) Formulate a conjecture as to the Laplace transform of f (t), tn with n a
positive integer.

15018 THE LAPLACE TRANSFORM: BASIC DEFINITIONS AND RESULTS


Solution.
(a) Using the two previous problems we find
( 
)

Z T
Z T
n st T
t
e
n
tn est dt = lim
+
tn1 est dt
L[tn ] = lim
T
T 0
s
s
0
0
Z T
n
n
tn1 est dt = L[tn1 ], s > 0.
= lim
s T 0
s
(b) We have
1
s2
2
2
L[t2 ] = L[t] = 3
s
s
3
6
L[t3 ] = L[t2 ] = 4
s
s
4
24
L[t4 ] = L[t3 ] = 5
s
s
5
120
L[t5 ] = L[t4 ] = 5 .
s
s
L[t] =

(c) By induction, one can easily show that for n = 0, 1, 2,


L[tn ] =

n!
sn+1

, s>0

From a table of integrals,


Z
sin u cos u
eu sin udu =eu
2 + 2
Z
cos u + sin u
eu cos udu =eu
2 + 2
Problem 18.13
Use the above integrals to find the Laplace transform of f (t) = cos t, if it
exists. If the Laplace transform exists, give the domain of F (s).
Solution.
We have
(
L[cos t] = lim est
T

s cos t + sin t
s2 + 2

T )
=
0

s2

s
, s>0
+ 2

151
Problem 18.14
Use the above integrals to find the Laplace transform of f (t) = sin t, if it
exists. If the Laplace transform exists, give the domain of F (s).
Solution.
We have
(
L[sin t] = lim est

s sin t + cos t
s2 + 2

T )
=
0

s2

, s>0
+ 2

Problem 18.15
Use the above integrals to find the Laplace transform of f (t) = cos (t 2),
if it exists. If the Laplace transform exists, give the domain of F (s).
Solution.
Using a trigonometric identity we can write f (t) = cos (t 2) = cos t cos 2+
sin t sin 2. Thus, using the previous two problems we find
L[cos (t 2)] =

s cos 2 + sin 2
, s>0
s2 + 2

Problem 18.16
Use the above integrals to find the Laplace transform of f (t) = e3t sin t, if it
exists. If the Laplace transform exists, give the domain of F (s).
Solution.
We have
3t

e(s3)t sin tdt


0
(

T )
(s

3)
sin
t
+
cos
t
= lim e(s3)t
T
(s 3)2 + 1
0

L[e sin t] = lim

1
, s>3
(s 3)2 + 1

Problem 18.17
Consider the function f (t) = tan t.
(a) Is f (t) continuous on 0 t < , discontinuous but piecewise continuous
on 0 t < , or neither?
(b) Are there fixed numbers a and M such that |f (t)| M eat for 0 t < ?

15218 THE LAPLACE TRANSFORM: BASIC DEFINITIONS AND RESULTS


Solution.
sin t
and this function is discontinuous at t = (2n +
(a) Since f (t) = tan t = cos
t

1) 2 . Since this function has vertical asymptotes there it is not piecewise


continuous.
(b) The graph of the function does not show that it can be bounded by
exponential functions. Hence, no such numbers a and M
Problem 18.18
t2
Consider the function f (t) = e2te +1 .
(a) Is f (t) continuous on 0 t < , discontinuous but piecewise continuous
on 0 t < , or neither?
(b) Are there fixed numbers a and M such that |f (t)| M eat for 0 t < ?
Solution.
t2
(a) Since et2 and e2t +1 are continuous everywhere, f (t) = e2te +1 is continuous
on 0 t < .
2
2
(b) Since e2t + 1 e2t + e2t = 2e2t , f (t) 21 et e2t = 12 et 2t . But for t 4
we have t2 2t >

t2
.
2

t2

Hence, f (t) > 12 e 2 . So f (t) is not of exponential order

Problem 18.19

3
.
Find L1 s2
Solution. 
1
Since L sa
=
L

1
,
sa
1

Problem 18.20
Find L1 s22 +
Solution.
Since L[t] =

1
,
s2

s > a we find

1
s+1

3
s2


= 3L

1
s2

= 3e2t , t 0

s > 0 and L


2
1
2+
s
s+1

1
sa

1
,
sa

s > a we find




1
1
1
= 2L
+L
s2
s+1
t
= 2t + e , t 0
1

19 Further Studies of Laplace


Transform
Problem 19.1
Use Table L to find L[2et + 5].
Solution.
L[2et + 5] = 2L[et ] + 5L[1] =

5
2
+ , s>1
s1 s

Problem 19.2
Use Table L to find L[e3t3 h(t 1)].
Solution.
L[e3t3 h(t 1)] = L[e3(t1) h(t 1)] = es L[e3t ] =

es
, s>3
s3

Problem 19.3
Use Table L to find L[sin2 t].
Solution.
1 cos 2t
1
1
L[sin t] = L[
] = (L[1]L[cos 2t]) =
2
2
2
2

1
s
2
s s + 4 2

Problem 19.4
Use Table L to find L[sin 3t cos 3t].
Solution.



sin 6t
1
3
L[sin 3t cos 3t] = L
= L[sin 6t] = 2
, s>0
2
2
s + 36
153


, s>0

154

19 FURTHER STUDIES OF LAPLACE TRANSFORM

Problem 19.5
Use Table L to find L[e2t cos 3t].
Solution.
L[e2t cos 3t] =

s2
, s>2
(s 2)2 + 9

Problem 19.6
Use Table L to find L[e4t (t2 + 3t + 5)].
Solution.
L[e4t (t2 +3t+5)] = L[e4t t2 ]+3L[e4t t]+5L[e4t ] =
Problem 19.7
Use Table L to find L1 [ s210
+
+25

3
5
2
+
+
, s>4
3
2
(s 4) (s 4) s 4

4
].
s3

Solution.
L1 [

s2

10
4
5
1
+
] = 2L1 [ 2
]+4L1 [
] = 2 sin 5t+4e3t , t 0
+ 25 s 3
s + 25
s3

Problem 19.8
5
Use Table L to find L1 [ (s3)
4 ].
Solution.
L1 [

5
5
3!
5
] = L1 [
] = e3t t3 , t 0
4
4
(s 3)
6
(s 3)
6

Problem 19.9
2s
Use Table L to find L1 [ es9 ].
Solution.
e2s
L [
] = e9(t2) h(t 2) =
s9
1

Problem 19.10
3s
Use Table L to find L1 [ e s2(2s+7)
].
+16

0,
e

9(t2)

0t<2
, t2

155
Solution.
We have
L1 [

3s
e3s (2s + 7)
s
7 1 e3s 4
1 e
]
=2L
[
]
+
L [ 2
]
s2 + 16
s2 + 16
4
s + 16
7
=2 cos 4(t 3)h(t 3) + sin 4(t 3)h(t 3), t 0
4

Problem 19.11
Graph the function f (t) = h(t 1) + h(t 3) for t 0, where h(t) is the
Heaviside step function, and use Table L to find L[f (t)].
Solution.
Note that

0, 0 t < 1
1, 1 t < 3
f (t) =

2,
t 3.

The graph of f (t) is shown below. Using Table L we find


es e3s
+
, s>0
L[f (t)] = L[h(t 1)] + L[h(t 3)] =
s
s

Problem 19.12
Graph the function f (t) = t[h(t 1) h(t 3)] for t 0, where h(t) is the
Heaviside step function, and use Table L to find L[f (t)].
Solution.
Note that

0, 0 t < 1
t, 1 t < 3
f (t) =

0,
t 3.

156

19 FURTHER STUDIES OF LAPLACE TRANSFORM

The graph of f (t) is shown below. Using Table L we find


L[f (t)] =L[(t 1)h(t 1) + h(t 1) (t 3)h(t 3) 3h(t 3)]
=L[(t 1)h(t 1)] + L[h(t 1)] L[(t 3)h(t 3)] 3L[h(t 3)]
es es e3s 3e3s
= 2 +
2
, s>1
s
s
s
s

Problem 19.13
Graph the function f (t) = 3[h(t 1) h(t 4)] for t 0, where h(t) is the
Heaviside step function, and use Table L to find L[f (t)].
Solution.
Note that

0, 0 t < 1
3, 1 t < 4
f (t) =

0,
t 4.

The graph of f (t) is shown below. Using Table L we find


L[f (t)] = 3L[h(t 1)] 3L[h(t 4)] =

3es 3e4s

, s>0
s
s

Problem 19.14
Graph the function f (t) = |2 t|[h(t 1) h(t 3)] for t 0, where h(t) is
the Heaviside step function, and use Table L to find L[f (t)].

157
Solution.
Note that
f (t) =

0,
0t<1
|2 t|, 1 t < 3

0,
t 3.

The graph of f (t) is shown below. Using Table L we find


L[f (t)] =(2 t)h(t 1) + 2(t 2)h(t 2) (t 2)h(t 3)
=L[(t 1)h(t 1) + h(t 1) + 2(t 2)h(t 2) (t 3)h(t 3) h(t 3)]
= L[(t 1)h(t 1)] + L[h(t 1)] + 2L[(t 2)h(t 2)]
L[(t 3)h(t 3)] L[h(t 3)]
es es 2e2s e3s e3s
= 2 +
+ 2 2
, s>0
s
s
s
s
s

Problem 19.15
Graph the function f (t) = h(2 t) for t 0, where h(t) is the Heaviside step
function, and use Table L to find L[f (t)].
Solution.
Note that

f (t) =

1, 0 t 2
0,
t > 2.

The graph of f (t) is shown below. From this graph we see that f (t) =
h(t) h(t 2). Using Table L we find
L[f (t)] = L[h(t)] L[h(t 2)] =

1 e2s
, s>0
s

158

19 FURTHER STUDIES OF LAPLACE TRANSFORM

Problem 19.16
Graph the function f (t) = h(t 1) + h(4 t) for t 0, where h(t) is the
Heaviside step function, and use Table L to find L[f (t)].
Solution.
Note that

1, 0 t < 1
2, 1 t 4
f (t) =

1,
t > 4.

The graph of f (t) is shown below. Using Table L we find


Z 4
1 + es e4s
es
+
est dt =
, s>0
L[f (t)] = L[h(t1)]+L[h(4t)] =
s
s
0

Problem 19.17
The graph of f (t) is given below. Represent f (t) as a combination of Heaviside step functions, and use Table L to calculate the Laplace transform of
f (t).

159
Solution.
From the graph we see that
f (t) = (t 2)h(t 2) (t 3)h(t 3) h(t 4).
Thus,
L[f (t)] = L[(t2)h(t2)]L[(t3)h(t3)]L[h(t4)] =

e2s e3s e4s


, s>0

s2
s

Problem 19.18
The graph of f (t) is given below. Represent f (t) as a combination of Heaviside step functions, and use Table L to calculate the Laplace transform of
f (t).

Solution.
From the graph we see that
f (t) = h(t 1) + h(t 2) 2h(t 3).
Thus,
L[f (t)] = L[h(t1)]2L[h(t3)]+L[h(t2)] =

es 2e3s + e2s
, s>0
s

Problem 19.19
Use Laplace transform technique to solve the initial value problem
y 0 + 4y = g(t), y(0) = 2
where

0, 0 t < 1
12, 1 t < 3
g(t) =

0,
t3

160

19 FURTHER STUDIES OF LAPLACE TRANSFORM

Solution.
Note first that g(t) = 12[h(t 1) h(t 3)] so that
L[g(t)] = 12L[h(t 1)] 12L[h(t 3)] =

12(es e3s )
, s > 0.
s

Now taking the Laplace transform of the DE and using linearity we find
L[y 0 ] + 4L[y] = L[g(t)].
But L[y 0 ] = sL[y] y(0) = sL[y] 2. Letting L[y] = Y (s) we obtain
sY (s) 2 + 4Y (s) = 12

es e3s
.
s

Solving for Y (s) we find


Y (s) =

2
es e3s
+ 12
.
s+4
s(s + 4)

But
L


2
= 2e4t
s+4

and
L





1
es e3s
1
1
s
3s

12
=3L
(e e )
s(s + 4)
s s+4
 s 
 3s 
 s 
 3s 
e
e
1 e
1 e
1
1
=3L
3L
3L
+ 3L
s
s
s+4
s+4
=3h(t 1) 3h(t 3) 3e4(t1) h(t 1) + 3e4(t3) h(t 3).

Hence,
y(t) = 2e4t +3[h(t1)h(t3)]3[e4(t1) h(t1)e4(t3) h(t3)], t 0

20 The Laplace Transform and


the Method of Partial Fractions
In Problems 20.1 - 20.4, give the form of the partial fraction expansion for
F (s). You need not evaluate the constants in the expansion. However, if the
denominator has an irreducible quadratic expression then use the completing
the square process to write it as the sum/difference of two squares.
Problem 20.1
F (s) =

s3 + 3s + 1
.
(s 1)3 (s 2)2

Solution.
F (s) =

A1
A2
A3
B1
B2
+
+
+
+
3
2
2
(s 1)
(s 1)
s 1 (s 2)
s2

Problem 20.2
F (s) =

s2 + 5s 3
.
(s2 + 16)(s 2)

Solution.
F (s) =

B1
A1 s + A2
+
2
s + 16
s2

Problem 20.3
F (s) =

s3 1
.
(s2 + 1)2 (s + 4)2
161

16220 THE LAPLACE TRANSFORM AND THE METHOD OF PARTIAL FRACTIONS


Solution.
F (s) =

A1 s + A2 A3 s + A4
B1
B2
+
+
+
(s2 + 1)2
s2 + 1
(s + 4)2 s + 4

Problem 20.4
F (s) =

s4 + 5s2 + 2s 9
.
(s2 + 8s + 17)(s 2)2

Solution.
F (s) =

A2
B1 s + B2
A1
+
+ 2
2
(s 2)
s 2 s + 8s + 17

Problemh20.5 i
1
.
Find L1 (s+1)
3
Solution.
h
i
1
1
Using Table L we find L
= 12 et t2 , t 0
(s+1)3
Problem20.6 
Find L1 s22s3
.
3s+2
Solution.
We factor the denominator and split the rational function into partial fractions:
A
B
2s 3
=
+
.
(s 1)(s 2)
s1 s2
Multiplying both sides by (s 1)(s 2) and simplifying to obtain
2s 3 = A(s 2) + B(s 1)
= (A + B)s 2A B.
Equating coefficients of like powers of s we obtain the system

A+B
= 2
2A B = 3.

163
Solving this system by elimination we find A = 1 and B = 1. Now finding
the inverse Laplace transform to obtain






1
1
2s 3
1
1
1
=L
+L
= et + e2t , t 0
L
(s 1)(s 2)
s1
s2
Problemh20.7 i
2
Find L1 4ss3+s+1
.
+s
Solution.
We factor the denominator and split the rational function into partial fractions:
A Bs + C
4s2 + s + 1
= + 2
.
2
s(s + 1)
s
s +1
Multiplying both sides by s(s2 + 1) and simplifying to obtain
4s2 + s + 1 = A(s2 + 1) + (Bs + C)s
= (A + B)s2 + Cs + A.
Equating coefficients of like powers of s we obtain A + B = 4, C = 1, A = 1.
Thus, B = 3. Now finding the inverse Laplace transform to obtain
 2



 


s
1
1 4s + s + 1
1 1
1
1
L
=L
+3L
+L
= 1+3 cos t+sin t, t 0.
s(s2 + 1)
s
s2 + 1
s2 + 1
Problemh20.8
i
2 +6s+8
Find L1 s4s+8s
2 +16 .
Solution.
We factor the denominator and split the rational function into partial fractions:
s2 + 6s + 8
B1 s + C1 B2 s + C2
=
+ 2
.
(s2 + 4)2
s2 + 4
(s + 4)2
Multiplying both sides by (s2 + 4)2 and simplifying to obtain
s2 + 6s + 8 = (B1 s + C1 )(s2 + 4) + B2 s + C2
= B1 s3 + C1 s2 + (4B1 + B2 )s + 4C1 + C2 .

16420 THE LAPLACE TRANSFORM AND THE METHOD OF PARTIAL FRACTIONS


Equating coefficients of like powers of s we obtain B1 = 0, C1 = 1, B2 = 6,
and C2 = 4. Now finding the inverse Laplace transform to obtain
 2







1
s
1
1 s + 6s + 8
1
1
1
L
=L
+ 6L
+ 4L
(s2 + 4)2
s2 + 4
(s2 + 4)2
(s2 + 4)2




1
t
1
= sin 2t + 6
sin 2t + 4
[sin 2t 2t cos 2t]
2
4
16
3
1
3
= t sin 2t + sin 2t t cos 2t, t 0
2
4
2
Problem 20.9
Use Laplace transform to solve the initial value problem
y 0 + 2y = 26 sin 3t, y(0) = 3.
Solution.
Taking the Laplace of both sides to obtain
L[y 0 ] + 2L[y] = 26L[sin 3t].
Using Table L the last equation reduces to

sY (s) y(0) + 2Y (s) = 26

3
2
s +9


.

Solving this equation for Y (s) we find


Y (s) =

78
3
+
.
s + 2 (s + 2)(s2 + 9)

Using the partial fraction decomposition we can write


1
A
Bs + C
=
+ 2
.
2
(s + 2)(s + 9)
s+2
s +9
Multipliying both sides by (s + 2)(s2 + 9) to obtain
1 = A(s2 + 9) + (Bs + C)(s + 2)
= (A + B)s2 + (2B + C)s + 9A + 2C.

165
Equating coefficients of like powers of s we find A + B = 0, 2B + C = 0, and
1
2
1
, B = 13
, and C = 13
.
9A + 2C = 1. Solving this system we find A = 13
Thus,




9
s
3
Y (s) =
6 2
+4 2
.
s+2
s +9
s +9
Finally,





1
s
3
1
1
y(t) = L [Y (s)] = 9L
6L
+ 4L
s+2
s2 + 9
s2 + 9
= 9e2t 6 cos 3t + 4 sin 3t, t 0
1

Problem 20.10
Use Laplace transform to solve the initial value problem
y 0 + 2y = 4t, y(0) = 3.
Solution.
Taking the Laplace of both sides to obtain
L[y 0 ] + 2L[y] = 4L[t].
Using Table L the last equation reduces to
sY (s) y(0) + 2Y (s) =

4
.
s2

Solving this equation for Y (s) we find


Y (s) =

4
3
+
.
s + 2 (s + 2)s2

Using the partial fraction decomposition we can write


1
A
Bs + C
=
+
.
2
(s + 2)s
s+2
s2
Multipliying both sides by (s + 2)s2 to obtain
1 = As2 + (Bs + C)(s + 2)
= (A + B)s2 + (2B + C)s + 2C.

16620 THE LAPLACE TRANSFORM AND THE METHOD OF PARTIAL FRACTIONS


Equating coefficients of like powers of s we find A + B = 0, 2B + C = 0, and
2C = 1. Solving this system we find A = 41 , B = 41 , and C = 12 . Thus,
Y (s) =

4
1
1
+ 2 2.
s+2 s
s

Finally,
 
 

1
1
1 1
1
L
+ 2L
y(t) = L [Y (s)] = 4L
s+2
s
s2
= 4e2t 1 + 2t, t 0
1

Problem 20.11
Use Laplace transform to solve the initial value problem
y 00 + 3y 0 + 2y = 6et , y(0) = 1, y 0 (0) = 2.
Solution.
Taking the Laplace of both sides to obtain
L[y 00 ] + 3L[y 0 ] + 2L[y] = 6L[et ].
Using Table L the last equation reduces to
s2 Y (s) sy(0) y 0 (0) + 3(sY (s) y(0)) + 2Y (s) =

6
.
s+1

Solving this equation for Y (s) we find


Y (s) =

s+5
6
s2 + 6s + 11
+
=
.
(s + 1)(s + 2) (s + 2)(s + 1)2
(s + 1)2 (s + 2)

Using the partial fraction decomposition we can write


s2 + 6s + 11
A
B
C
=
+
+
.
2
(s + 2)(s + 1)
s + 2 s + 1 (s + 1)2
Multipliying both sides by (s + 2)(s + 1)2 to obtain
s2 + 6s + 11 = A(s + 1)2 + B(s + 1)(s + 2) + C(s + 2)
= (A + B)s2 + (2A + 3B + C)s + A + 2B + 2C.

167
Equating coefficients of like powers of s we find A + B = 1, 2A + 3B + C = 6,
and A + 2B + 2C = 11. Solving this system we find A = 3, B = 2, and
C = 6. Thus,
2
6
3

+
Y (s) =
.
s + 2 s + 1 (s + 1)2
Finally,





1
1
1
1
1
2L
+ 6L
y(t) = L [Y (s)] = 3L
s+2
s+1
(s + 1)2
= 3e2t 2et + 6tet , t 0
1

Problem 20.12
Use Laplace transform to solve the initial value problem
y 00 + 4y = cos 2t, y(0) = 1, y 0 (0) = 1.
Solution.
Taking the Laplace of both sides to obtain
L[y 00 ] + 4L[y] = L[cos 2t].
Using Table L the last equation reduces to
s2 Y (s) sy(0) y 0 (0) + 4Y (s) =

s2

s
.
+4

Solving this equation for Y (s) we find


Y (s) =

s+1
s
+ 2
.
2
s + 4 (s + 4)2

Using Table L again we find








s
1 1
2
s
1
1
y(t) = L
+ L
+L
s2 + 4
2
s2 + 4
(s2 + 4)2
1
t
= cos 2t + sin 2t + sin 2t, t 0
2
4
Problem 20.13
Use Laplace transform to solve the initial value problem
y 00 2y 0 + y = e2t , y(0) = 0, y 0 (0) = 0.

16820 THE LAPLACE TRANSFORM AND THE METHOD OF PARTIAL FRACTIONS


Solution.
Taking the Laplace of both sides to obtain
L[y 00 ] 2L[y 0 ] + L[y] = L[e2t ].
Using Table L the last equation reduces to
s2 Y (s) sy(0) y 0 (0) 2sY (s) + 2y(0) + Y (s) =

1
.
s2

Solving this equation for Y (s) we find


Y (s) =

1
.
(s 1)2 (s 2)

Using the partial fraction decomposition, we can write


Y (s) =

A
B
C
+
+
.
s 1 (s 1)2 s 2

Multipliying both sides by (s 2)(s 1)2 to obtain


1 = A(s 1)(s 2) + B(s 2) + C(s 1)2
= (A + C)s2 + (3A + B 2C)s + 2A 2B + C.
Equating coefficients of like powers of s we find A+C = 0, 3A+B2C = 0,
and 2A 2B + C = 1. Solving this system we find A = 1, B = 1, and
C = 1. Thus,
1
1
1
Y (s) =

+
.
2
s 1 (s 1)
s2
Finally,






1
1
1
1
1
1
1
L
+L
y(t) = L [Y (s)] = L
s1
(s 1)2
s2
t
t
2t
= e te + e , t 0
Problem 20.14
Use Laplace transform to solve the initial value problem
y 00 + 9y = g(t), y(0) = 1, y 0 (0) = 3
where


g(t) =

6, 0 t <
0, t < .

169
Solution.
Taking the Laplace of both sides to obtain
L[y 00 ] + 9L[y] = L[g(t)] = 6L[h(t) h(t )].
Using Table L the last equation reduces to
s2 Y (s) sy(0) y 0 (0) + 9Y (s) =

6 6es

.
s
s

Solving this equation for Y (s) we find


Y (s) =

s+3
6
+
(1 es ).
2
2
s + 9 s(s + 9)

Using the partial fraction decomposition, we can write


A Bs + C
6
= + 2
.
+ 9)
s
s +9

s(s2

Multipliying both sides by s(s2 + 9) to obtain


6 = A(s2 + 9) + (Bs + C)s
= (A + B)s2 + Cs + 9A.
Equating coefficients of like powers of s we find A + B = 0, C = 0, and
9A = 6. Solving this system we find A = 32 , B = 32 , and C = 0. Thus,


21 2 s
s
3
s
Y (s) = 2
+
+ (1 e )

.
s + 9 s2 + 9
3 s 3 s2 + 9
Finally,
2
2
y(t) = L1 [Y (s)] = cos 3t + sin 3t + (1 cos 3t) (1 cos 3(t ))h(t )
3
3
2
2
= cos 3t + sin 3t + (1 cos 3t) (1 + cos 3t)h(t ), t 0
3
3
Problem 20.15
Determine the constants , , y0 , and y00 so that Y (s) =
transform of the solution to the initial value problem

2s1
s2 +s+2

y 00 + y 0 + y = 0, y(0) = y0 , y 0 (0) = y00 .

is the Laplace

17020 THE LAPLACE TRANSFORM AND THE METHOD OF PARTIAL FRACTIONS


Solution.
Taking the Laplace transform of both sides we find
s2 Y (s) sy0 y00 + sY (s) y0 + Y (s) = 0.
Solving for Y (s) we find
Y (s) =

sy0 + (y00 + y0 )
2s 1
=
.
s2 + s +
s2 + s + 2

By identification we find = 1, = 2, y0 = 2, and y00 = 3

21 Laplace Transforms of
Periodic Functions
Problem 21.1
Find the Laplace transform of the periodic function whose graph is shown.

Solution.
The function is of period T = 2. Thus,
Z

3
0

st

Z
dt +

2
st

e
1

3
dt = est
s

1

est

s
0


2
1

1
= (3 2es e2s ).
s

Hence,
L[f (t)] =

3 2es e2s
s(1 e2s )

Problem 21.2
Find the Laplace transform of the periodic function whose graph is shown.
171

172

21 LAPLACE TRANSFORMS OF PERIODIC FUNCTIONS

Solution.
The function is of period T = 4. Thus,

1  st 3
Z 1
Z 3
e
2 st
1
st
st
2
e dt +
e dt = e

= (2 es e3s ).
s
s 1 s
0
1
0
Hence,
L[f (t)] =

2 es e3s
s(1 e4s )

Problem 21.3
Find the Laplace transform of the periodic function whose graph is shown.

Solution.
The function is of period T = 2. Thus,
Z 2
Z 2
Z 2
st
st
(t 1)e dt =
te dt
est dt
1
1
1

2
st
t st e
est
= e 2 +
s
s
s 1
s
e
= 2 [(1 + s)es 1]
s

173
Hence,
L[f (t)] =

es
[1 (s + 1)es ]
2
2s
s (1 e )

Problem 21.4
Find the Laplace transform of the periodic function whose graph is shown.

Solution.
The function is of period T = 2. Thus,
Z

2
st

te
0

1
dt = 2 (st + 1)est
s

2
=
0

1
[(2s + 1)e2s 1].
2
s

Hence,
L[f (t)] =

s2 (1

1
[1 (2s + 1)e2s ]
2s
e )

Problem 21.5
State the period of the function f (t) and find its Laplace transform where

f (t) =

sin t,

0,

0t<
f (t + 2) = f (t), t 0.
t < 2

Solution.
The graph of f (t) is shown below.

174

21 LAPLACE TRANSFORMS OF PERIODIC FUNCTIONS

The function f (t) is of period T = 2. The Laplace transform of f (t) is


R
sin test dt
L[f (t)] = 0
1 e2s
Using integration by parts twice we find
Z
est
(cos t + s sin t)
sin test dt =
1 + s2
Thus,
Z

st

sin te
0



est
dt =
(cos t + s sin t)
1 + s2
0
1
es
+
=
1 + s2 1 + s2
1 + es
=
1 + s2

Hence,
L[f (t)] =

1 + es
(1 + s2 )(1 e2s )

Problem 21.6
State the period of the function f (t) = 1 et , 0 t < 2, f (t + 2) = f (t),
and find its Laplace transform.
Solution.
The graph of f (t) is shown below

175

The function is periodic of period T = 2. Its Laplace transform is


R2
(1 et )est dt
.
L[f (t)] = 0
1 e2s
But
 st 2  (s+1)t 2
Z 2
e
1
e
1
t st
+
= (1e2s )
(1e )e dt =
(1e2(s+1) ).
s 0
s+1 0 s
s+1
0
Hence,
L[f (t)] =

1
1 e2(s+1)

s (s + 1)(1 e2s )

Problem 21.7
Using Example 21.3 find
L


es
s2 s
+
.
s3
s(1 es )

Solution.
Note first that
es
s2 s
1
+
=
3
s
s
s(1 e )
s

ses
1

s2 s2 (1 es )

Using Example 22.3, we find


g(t) = 1 f (t)
where f (t) is the sawtooth function shown below


.

176

21 LAPLACE TRANSFORMS OF PERIODIC FUNCTIONS

Problem 21.8
Consider the initial value problem
ay 00 + by 0 + cy = f (t), y(0) = y 0 (0) = 0, t > 0.
Suppose that the transfer function of this system is given by (s) =
(a) What are the constants a, b, and c?
(b) If f (t) = et , determine F (s), Y (s), and y(t).

1
.
2s2 +5s+2

Solution.
(a) Taking the Laplace transform of both sides we find as2 Y (s) + bsY (s) +
cY (s) = F (s) or
(s) =

1
1
Y (s)
= 2
= 2
.
F (s)
as + bs + c
2s + 5s + 2

By identification we find a = 2, b = 5, and c = 2.


1
(b) If f (t) = et then F (s) = L[et ] = s+1
. Thus,
Y (s) = (s)F (s) =

1
.
(s + 1)(2s2 + 5s + 2)

Using partial fraction decomposition


1
A
B
C
=
+
+
(s + 1)(2s + 1)(s + 2)
s + 1 2s + 1 s + 2

177
Multiplying both sides by s + 1 and setting s = 1 we find A = 1. Next,
multiplying both sides by 2s+1 and setting s = 12 we find B = 34 . Similarly,
multiplying both sides by s + 2 and setting s = 2 we find C = 13 . Thus,






1
1 1
1
1
2 1
1
+ L
+ L
y(t) = L
s+1
3
3
s+2
s + 12
2 t 1
= et + e 2 + e2t , t 0
3
3
Problem 21.9
Consider the initial value problem
ay 00 + by 0 + cy = f (t), y(0) = y 0 (0) = 0, t > 0
Suppose that an input f (t) = t, when applied to the above system produces
the output y(t) = 2(et 1) + t(et + 1), t 0.
(a) What is the system transfer function?
(b) What will be the output if the Heaviside unit step function f (t) = h(t)
is applied to the system?
Solution.
(a) Since f (t) = t we find F (s) = s12 . Aslo, Y (s) = L[y(t)] = L[2et 2 +
Y (s)
2
1
1
1
1
tet + t] = s+1
2s + (s+1)
2 + s2 = s2 (s+1)2 . But (s) = F (s) = (s+1)2 .
1
(b) If f (t) = h(t) then F (s) = 1s and Y (s) = (s)F (s) = s(s+1)
2 . Using
partial fraction decomposition we find
A
B
C
1
= +
+
2
s(s + 1)
s
s + 1 (s + 1)2
1 =A(s + 1)2 + Bs(s + 1) + Cs
1 =(A + B)s2 + (2A + B + C)s + A
Equating coefficients of like powers of s we find A = 1,
C = 1. Therefore,
Y (s) =

1
1
1

s s + 1 (s + 1)2

and
y(t) = L1 [Y (s)] = 1 et tet , t 0

B = 1, and

178

21 LAPLACE TRANSFORMS OF PERIODIC FUNCTIONS

Problem 21.10
Consider the initial value problem
y 00 + y 0 + y = f (t), y(0) = y 0 (0) = 0,
where
f (t) =

1,

0t1
f (t + 2) = f (t)

1, 1 < t < 2

(a) Determine the system transfer function (s).


(b) Determine Y (s).
Solution.
(a) Taking the Laplace transform of both sides we find
s2 Y (s) + sY (s) + Y (s) = F (s)
so that
(s) =

Y (s)
1
= 2
.
F (s)
s +s+1

(b) But
Z

f (t)e

st

dt
est dt
0
1
 st 1  st 2
e
e

=
s 0
s 1
1
1
= (1 es ) + (e2s es )
s
s
(1 es )2
=
s

dt =

st

Hence,
F (s) =

(1 es )2
(1 es )
=
s(1 e2s )
s(1 + es )

and
Y (s) = (s)F (s) =

(1 es )
s(1 + es )(s2 + s + 1)

179
Problem 21.11
Consider the initial value problem
y 000 4y = et + t, y(0) = y 0 (0) = y 00 (0) = 0.
(a) Determine the system transfer function (s).
(b) Determine Y (s).
Solution.
(a) Taking Laplace transform of both sides we find
s3 Y (s) 4Y (s) = F (s).
Thus,
(s) =

1
Y (s)
= 3
.
F (s)
s 4

(b) We have
F (s) = L[et + t] =

1
1
s2 + s 1
+ 2 =
.
s1 s
(s 1)s2

Hence,
s2 + s 1
Y (s) = 2
s (s 1)(s3 4)
Problem 21.12
Consider the initial value problem
y 00 + by 0 + cy = h(t), y(0) = y0 , y 0 (0) = y00 , t > 0.
Suppose that L[y(t)] = Y (s) =
and y00 .

s2 +2s+1
.
s3 +3s2 +2s

Determine the constants b, c, y0 ,

Solution.
Take the Laplace transform of both sides to obtain
1
s2 Y (s) sy0 y00 + bsY (s) by0 + cY (s) = .
s
Solve to find
s2 y0 + s(y00 + by0 ) + 1
s3 + bs2 + cs
2
s + 2s + 1
.
= 3
s + 3s2 + 2s
By comparison we find b = 3, c = 2, y0 = 1, and y00 + by0 = 2 or y00 = 1
Y (s) =

180

21 LAPLACE TRANSFORMS OF PERIODIC FUNCTIONS

22 Convolution Integrals
Problem 22.1
Consider the functions f (t) = g(t) = h(t), t 0 where h(t) is the Heaviside
unit step function. Compute f g in two different ways.
(a) By directly evaluating the integral.
(b) By computing L1 [F (s)G(s)] where F (s) = L[f (t)] and G(s) = L[g(t)].
Solution.
(a) We have
Z
(f g)(t) =

Z
f (t s)g(s)ds =

Z
h(t s)h(s)ds =

(b) Since F (s) = G(s) = L[h(t)] =


L1 [ s12 ] = t, t 0

1
s

ds = t, t 0.
0

we have (f g)(t) = L1 [F (s)G(s)] =

Problem 22.2
Consider the functions f (t) = et and g(t) = e2t , t 0. Compute f g in
two different ways.
(a) By directly evaluating the integral.
(b) By computing L1 [F (s)G(s)] where F (s) = L[f (t)] and G(s) = L[g(t)].
Solution.
(a) We have
t

(f g)(t) =

e(ts) e2s ds

f (t s)g(s)ds =
0
t

=e

t
3s


ds =

0
t

2t

e e
=
3

, t 0.
181

(t3s)

t
0

182

22 CONVOLUTION INTEGRALS

1
1
(b) Since F (s) = L[et ] = s1
and G(s) = L[e2t ] = s+2
we find (f g)(t) =
1
1
1
L [F (s)G(s)] = L [ (s1)(s2) ]. Using partial fractions decomposition we
find
1 1
1
1
= (

).
(s 1)(s + 2)
3 s1 s+2

Thus,
1
(f g)(t) = L [F (s)G(s)] =
3
1



1
1
et e2t
1
1
L [
]L [
] =
,t 0
s1
s+2
3

Problem 22.3
Consider the functions f (t) = sin t and g(t) = cos t, t 0. Compute f g in
two different ways.
(a) By directly evaluating the integral.
(b) By computing L1 [F (s)G(s)] where F (s) = L[f (t)] and G(s) = L[g(t)].
Solution.
(a) Using the trigonometric identity 2 sin p cos q = sin (p + q) + sin (p q) we
find that 2 sin (t s) cos s = sin t + sin (t 2s). Hence,
Z t
Z t
(f g)(t) =
f (t s)g(s)ds =
sin (t s) cos sds
0
0
Z
Z t
1 t
sin (t 2s)ds]
= [ sin tds +
2 0
0
Z
t sin t 1 t
=
+
sin udu
2
4 t
t sin t
=
.t 0.
2
(b) Since F (s) = L[sin t] =

1
s2 +1

and G(s) = L[cos t] =

(f g)(t) = L1 [F (s)G(s)] = L1 [

(s2

s
s2 +1

we find

s
t
] = sin t, t 0
2
+ 1)
2

Problem 22.4
Compute and graph f g where f (t) = h(t) and g(t) = t[h(t) h(t 2)].
Solution.
Since f (t) = h(t), F (s) =

1
.
s

Similarly, since g(t) = th(t) th(t 2) =

183
th(t) (t 2)h(t 2) 2h(t 2), G(s) =
2s
2s
1
e s3 2es2 . It follows that
s3
(f g)(t) =

1
s2

2s

2s

e s2 2e s . Thus, F (s)G(s) =

t2 (t 2)2

h(t 2) 2(t 2)h(t 2), t 0.


2
2

The graph of (f g)(t) is given below

Problem 22.5
Compute and graph f g where f (t) = h(t) h(t 1) and g(t) = h(t 1)
2h(t 2).
Solution.
Since f (t) = h(t) h(t 1), F (s) = 1s
s
2s
1) 2h(t 2), G(s) = e s 2e s . Thus,

es
.
s

Similarly, since g(t) = h(t

es 3e2s + 2e3s
s2
s
e
e2s
e3s
= 2 3 2 +2 2
s
s
s

F (s)G(s) =

It follows that
(f g)(t) = (t 1)h(t 1) 3(t 2)h(t 2) + 2(t 3)h(t 3), t 0.
The graph of (f g)(t) is given below

184

22 CONVOLUTION INTEGRALS

Problem 22.6
Compute t t t.
Solution.
By the convolution theorem we have L[t t t] = (L[t])3 =
 
5
t5
t t t = L1 s16 = t5! = 120
,t 0


1 3
2
s

1
.
s6

Hence,

Problem 22.7
Compute h(t) et e2t .
Solution.
By the convolution theorem we have L[h(t)et e2t ] = L[h(t)]L[et ]L[e2t ] =
1
1 1 . Using the partial fractions decomposition we can write
s s+1 s+2
1
1
1
1
1
=

+
.
s(s + 1)(s + 2)
2s s + 1 2 s + 2
Hence,
h(t) et e2t =

1
1
et + e2t , t 0
2
2

Problem 22.8
Compute t et et .
Solution.
By the convolution theorem we have L[t et et ] = L[t]L[et ]L[et ] =
1
1 . Using the partial fractions decomposition we can write
s+1 s1
s2 (s

1
1
1
1
1
1
= 2 +

.
+ 1)(s 1)
s
2 s1 2 s+1

1
s2

185
Hence,
t

te

et et
e = t +
,t 0
2
2
t

Problem 22.9

n f unctions

z
}|
{
Suppose it is known that h(t) h(t) h(t) = Ct8 . Determine the constants C and the positive integer n.
Solution.

n f unctions

}|
{
z
We know that L[h(t) h(t) h(t)] = (L[h(t)])n =
tn1
= Ct8 . It follows that n = 9 and C = 8!1
(n1)!

1
sn

so that L1 [ s1n ] =

Problem 22.10
Use Laplace transform to solve for y(t) :
Z t
sin (t )y()d = t2 .
0

Solution.
Note that the given equation reduces to sin t y(t) = t2 . Taking Laplace
2
transform of both sides we find sY2(s)
= s23 . This implies Y (s) = 2(ss3+1) =
+1
2
+ s23 . Hence, y(t) = L1 [ 2s + s23 ] = 2 + t2 , t 0
s

You might also like